Vision Ias Prelims 2020 Test 19 Solution.pdf

  • Uploaded by: kanchan singh
  • 0
  • 0
  • January 2021
  • PDF

This document was uploaded by user and they confirmed that they have the permission to share it. If you are author or own the copyright of this book, please report to us by using this DMCA report form. Report DMCA


Overview

Download & View Vision Ias Prelims 2020 Test 19 Solution.pdf as PDF for free.

More details

  • Words: 22,141
  • Pages: 33
Loading documents preview...
https://t.me/UPSC_PDF

https://upscpdf.com

https://t.me/UPSC_PDF

VISIONIAS www.visionias.in ANSWERS & EXPLANATIONS GENERAL STUDIES (P) TEST – 2981 (2020) Q 1.A •







Joint Forest Management (JFM) is an approach and program initiated in the context of the National Forest Policy of 1988 wherein state forest departments support local forest-dwelling and forest fringe communities to protect and manage forests and share the costs and benefits from the forests with them. Here, there is no legal sanction for JFM. Hence statement 2 is not correct. Accordingly, JFM tries to harness the strengths and energy of local rural communities for protecting and managing forests through JFM Committees/ Eco Development Committees and helps to meet their needs for subsistence and livelihood as well as generates local environmental services. Hence statement 1 is correct. Membership for a JFMC is open for all members of the gram sabha – i.e. voting adults. (in some states membership is open to one male and one female member of each household). The Executive Committee is responsible for managing the overall operations of the JFMC. The JFMC member will elect the Executive Committee from the JFMC/ EDC members, or as defined in the state guidelines. JFM has the potential to meet local subsistence needs, of fuelwood, fodder, other non-timber forest produce, small timber and timber etc., to provide livelihood through sale of produce, while at the same time, preventing degradation of the forests that provide local, national and global environmental benefits.

Q 2.C •







On 29 January 2000, the Conference of the Parties to the Convention on Biological Diversity adopted a supplementary agreement to the Convention known as the Cartagena Protocol on Biosafety. It is an international treaty governing the movements of living modified organisms (LMOs) resulting from modern biotechnology from one country to another. Hence option 1 is correct. Under the Protocol, the Advance Informed Agreement (AIA) procedure applies to the first intentional transboundary movement of an LMO for intentional introduction into the environment of the Party of import. The advance informed agreement or AIA procedure is designed to ensure that before an LMO is imported into a country for the first time for intentional introduction into the environment, the Party of import is notified about the proposed import, receives full information about the LMO and its intended use and has an opportunity to assess the risks associated with that LMO and to decide whether or not to allow the import. Hence option 4 is correct. The Nagoya Protocol on Access to Genetic Resources and the Fair and Equitable Sharing of Benefits Arising from their Utilization (ABS) to the Convention on Biological Diversity is a supplementary agreement to the Convention on Biological Diversity. It provides a transparent legal framework for the effective implementation of one of the three objectives of the CBD: the fair and equitable sharing of benefits arising out of the utilization of genetic resources, thereby contributing to the conservation and sustainable use of biodiversity. Hence, option 3 is correct. The Protocol for the Prohibition of the Use in War of Asphyxiating, Poisonous or other Gases, and of Bacteriological Methods of Warfare, usually called the Geneva Protocol, is a treaty prohibiting the use of chemical and biological weapons in international armed conflicts. Hence option 2 is not correct.

Q 3.D •



1

Forest Right Act 2006: It provides for a rights-based, democratic and decentralized governance of forests. Rights recognized under FRA. Individual forest rights (IFR) to legally hold forestlands that the forest dwelling communities have been residing on and cultivating prior to 13 December 2005. Community rights (CRs) of ownership, use and disposal of ‘minor forest produce’, also known as nontimber forest produce (NTFP). CRs include rights of grazing, collection of firewood, fish and other such www.visionias.in

Join For More Update : - https://upscpdf.com

©Vision IAS

https://t.me/UPSC_PDF





https://upscpdf.com

https://t.me/UPSC_PDF

products from water bodies, as well as rights to biodiversity and intellectual property, including those related to traditional knowledge. Hence, statement 2 is not correct. Community forest resource (CFR) rights under Section 3(1)(i) to protect, regenerate, conserve or manage forest resources for sustainable use, providing for community governance of forests. Hence, statement 1 is not correct. CFR rights is the most empowering provision of the Act because it restores gram sabha’s control over governance of forests from the forest department, thereby democratising the country’s colonial forest governance as a whole. CFR management committees (CFRMCs) are created by Gram Sabha, which are expected to prepare a conservation and management plan for community forest resources in order to sustainably and equitably manage CFR areas.

Q 4.B •







Statement 1 is not correct: National Afforestation and Eco-Development Board (NAEB) was set up in 1992 under the Ministry of Environment, Forest and Climate Change. It wasn't set up under the provisions of the Forest Conservation Act, 1980. Statement 2 is correct: It is responsible for promoting afforestation, tree planting, ecological restoration and eco-development activities in the country, with special attention to the degraded forest areas and lands adjoining the forest areas, national parks, sanctuaries and other protected areas as well as the ecologically fragile areas like the Western Himalayas, Aravallis, Western Ghats, etc. National Mission for a Green India or the commonly called Green India Mission (GIM), is one of the eight Missions outlined under India’s action plan (National Action Plan on Climate Change) for addressing the challenge of climate change. It is aimed at protecting, restoring and enhancing India’s diminishing forest cover and responding to climate change by a combination of adaptation and mitigation measures. It has the broad objective of both increasing the forest and tree cover by 5 million ha, as well as increasing the quality of the existing forest and tree cover in another 5 million ha of forest/non-forest lands in 10 years. Statement 3 is not correct: NAEB doesn't act as the nodal agency for the implementation of the Green India Mission at the national level. At the national level, the Mission is set up as an autonomous Society under the aegis of the Ministry of Environment, Forest and Climate Change (MoEFCC) to facilitate smooth implementation of the Mission. The Governing Council of the Society, Chaired by the Minister for Environment and Forests, Government of India and drawing upon cross-sectoral representation, provides the overall guidance. In addition, the National Executive Council (NEC) chaired by the Secretary of Ministry of Environment and Forest has the overall responsibility for the Mission which approves the Perspective Plans & Annual Plan of Operations (APOs) submitted by States.

Q 5.B •







2

Statement 1 is not correct: According to the Citizenship Amendment Act, 2019 (CAA), members of the Hindu, Christian, Sikh, Buddhist and Zoroastrian communities who have come from Pakistan, Afghanistan, and Bangladesh till December 31, 2014, and facing religious persecution there will not be treated as illegal immigrants but given Indian citizenship. It also provides for "Citizenship by naturalisation". One of the qualifications is that the person must have resided in India or been in central government service for the last 12 months and at least 11 years of the preceding 14 years. The Act creates an exception for Hindus, Sikhs, Buddhists, Jains, Parsis and Christians from Afghanistan, Bangladesh and Pakistan, with regard to this qualification. Only for these groups of persons, the 11 years’ requirement will be reduced to six years. The Act also amends the provisions on registration of Overseas Citizens of India (OCI). OCI cardholders are foreigners who are persons of Indian origin. For example, they may have been former Indian citizens or children of current Indian citizens. An OCI enjoys benefits such as the right to travel to India without a visa, or to work and study here. Earlier, the government may cancel a person’s OCI registration on various grounds specified in the 1955 Act. In case of a cancellation, an OCI residing in India may be required to leave the country. The 2019 Act adds another ground for cancelling OCI registration — the violation of any law notified by the central government. Thus the Act provides Wider discretion to the government to cancel OCI registration. Statement 2 is correct: The Act adds that the provisions on citizenship for illegal migrants will not apply to the tribal areas of Assam, Meghalaya, Mizoram, or Tripura, as included in the Sixth Schedule to the Constitution. These tribal areas include Karbi Anglong (in Assam), Garo Hills (in Meghalaya), Chakma District (in Mizoram), and Tripura Tribal Areas District. It will also not apply to the areas under the Inner Line” under the Bengal Eastern Frontier Regulation, 1873. The Inner Line Permit regulates visit of Indians to Arunachal Pradesh, Mizoram, and Nagaland. www.visionias.in

Join For More Update : - https://upscpdf.com

©Vision IAS

https://t.me/UPSC_PDF

https://upscpdf.com

https://t.me/UPSC_PDF

Q 6.A •







• • •



The Banni grassland in the Kachchh District of Gujarat is an area of 2500 km2and is often referred to as one of the largest tropical grasslands in Asia. The Banni is home to great biological diversity, having 37 grass species, 275 bird species, and domesticated animals, like Banni Buffalo, Kankrej Cattle, Sheep & Goat, Horses and Camel, as well as wildlife. The Maldhari community is a tribe of herdsmen in the state of Gujarat. The name Maldhari means the owner of goods - in this case, goods referring to cattle. The Maldharis have lived in the Gir National Park, in the Banni Grasslands Reserve area, for the past thousand years. They have co-existed with the lions, which the Gir National Park was created to preserve, for these thousand years. In the early 1960s, the Gujarat Forest Department planted Prosopis juliflora in 31,550 ha (~315 km2) of the Banni with a stated objective of minimizing the perceived threats of salinity ingress and desertification. Over the past 50 years, Prosopis has spread from 315 km2 to close to 1500 km2 of the Banni. This expansion is thought to have taken place at the expense of palatable and perennial grass species. Chir Batti refers to an unexplained light phenomenon that occurs on dark nights in the Banni grasslands of Gujarat. Local villagers refer to the light as Chir Batti in their Kutchhi-Sindhi language, with Chir meaning ghost and Batti meaning light. Recently built dams, overgrazing have also taken their toll on both the biodiversity of the grassland and Maldharis. Bugyals are alpine pasture lands, or meadows, in higher elevation range between 3,300 metres and 4,000 metres of the Himalayas in the Indian state of Uttarakhand. Khajjiar is a small plateau in Chamba district Himachal Pradesh and surrounded by green meadows and dense forests. Grasslands of Khajjiar is covered by dense deodar and fir forest lies in the path of the Ravi River and offers numerous trekking opportunities. Sholas are the local name for patches of stunted tropical montane forest found in valleys amid rolling grassland in the higher montane regions of South India.

Q 7.B •

• •



Biosphere Reserves (BRs) are representative parts of natural and cultural landscapes extending over large area of terrestrial or coastal/marine ecosystems or a combination thereof and representative examples of bio-geographic zones/provinces. The core area of biosphere reserve should be typical of a bio-geographical unit and large enough to sustain viable populations representing all trophic levels in the ecosystem. The UNESCO has introduced the designation 'Biosphere Reserve' for natural areas to minimize conflict between development and conservation. BRs are nominated by national government which meet a minimal set of criteria and adhere to minimal set of conditions for inclusion in the world network of Biosphere reserves under the Man and Biosphere Reserve Programme of UNESCO. Thus only only 10 of the 18 biosphere reserves are covered under MAB (Man and the Biosphere Programme). At present, there is no comprehensive legislation dealing with all aspect of biosphere. Only few aspects have legal backing e.g. the core areas like National park or wildlife sanctuaries of reserve are governed under Wildlife Protection Act, 1972.

Q 8.D •







3

The International whaling commission (IWC) was set up under the International Convention for the Regulation of Whaling which was signed in Washington DC on 2nd December 1946. The preamble to the Convention states that its purpose is to provide for the proper conservation of whale stocks and thus make possible the orderly development of the whaling industry. Hence, statement 1 is correct. An integral part of the Convention is its legally binding 'Schedule.' The Schedule sets out specific measures that the IWC has collectively decided are necessary in order to regulate whaling and conserve whale stocks. Hence, statement 2 is not correct. These measures include catch limits (which may be zero as it the case for commercial whaling) by species and area, designating specified areas as whale sanctuaries, protection of calves and females accompanied by calves, and restrictions on hunting methods. Unlike the Convention, the Schedule can be amended and updated when the Commission meets (a change requires at least three quarters majority agreement). Membership of the IWC is open to any country in the world that formally adheres to the 1946 Convention. Each member country is known as a Contracting Government and represented by a Commissioner, who is assisted by experts and advisers. India is a member to the convention. Hence, statement 3 is correct. The Chair and Vice-chair are elected from among the Commissioners and usually serve for four years, first as Vice-chair for two years, and then as Chair. www.visionias.in ©Vision IAS

Join For More Update : - https://upscpdf.com

https://t.me/UPSC_PDF

https://upscpdf.com

https://t.me/UPSC_PDF

Q 9.C •





Biodiversity Finance Initiative (BIOFIN) is a global partnership addressing the biodiversity finance challenge in a comprehensive manner. The Initiative provides an innovative methodology enabling countries to measure their current biodiversity expenditures, assess their financial needs in the medium term and identify the most suitable finance solutions to bridge their national biodiversity finance gaps. United Nations Development Programme (UNDP) in October 2012 launched the Biodiversity Finance Initiative – BIOFIN, as a new global partnership seeking to address the biodiversity finance challenge in a comprehensive manner – building a sound business case for increased investment in the management of ecosystems and biodiversity. BIOFIN is managed by the UNDP Ecosystems and Biodiversity Programme, in partnership with the European Union and the Governments of Germany and Switzerland. India is also a part of this initiative. For the purpose of implementing BIOFIN project in India, the Ministry of Environment, Forest and Climate Change has identified National Biodiversity Authority (NBA) as the host organisation.

Q 10.D •





CITES (the Convention on International Trade in Endangered Species of Wild Fauna and Flora) is an international agreement between governments. Its aim is to ensure that international trade in specimens of wild animals and plants does not threaten their survival. Roughly 5,800 species of animals and 30,000 species of plants are protected by CITES against over-exploitation through international trade. They are listed in the three CITES Appendices. The species are grouped in the Appendices according to how threatened they are by international trade. They include some whole groups, such as primates, cetaceans (whales, dolphins and porpoises), sea turtles, parrots, corals, cacti and orchids. There are 155 bird species in Appendix I of CITES. As an environmental treaty under the aegis of the United Nations Environment Programme, Convention on the Conservation of Migratory Species of Wild Animals (CMS) provides a global platform for the conservation and sustainable use of migratory animals and their habitats. Migratory species threatened with extinction are listed on Appendix I of the Convention. There are about 300 avian/ bird species covered by the CMS. The Ramsar Convention on Wetlands of International Importance especially as Waterfowl Habitat is an international treaty for the conservation and sustainable use of wetlands. It is also known as the Convention on Wetlands. Under Article 4 of the convention, each Contracting Party shall promote the conservation of wetlands and waterfowl by establishing nature reserves on wetlands, whether they are included in the List or not, and provide adequately for their wardening.

Q 11.D •









4

Peats are a heterogeneous mixture of plant material (vascular plants, mosses and humus) that had accumulated in a water-saturated area and are only partially decomposed due to absence of oxygen. The natural areas covered by peat are called peatlands. Various types of peat are – swamp forests, fens, bogs or mires. They form where climate, bedrock and relief create an area with permanent water saturation i.e. either in shallow water over layers of lake sediments or directly on mineral soil. Peatlands are the largest natural terrestrial carbon store; the area covered by near-natural peatland worldwide sequesters 0.37 gigatonnes of carbon dioxide a year – storing more carbon than all other vegetation types in the world combined. By conserving, protecting and restoring peatlands globally, we can reduce emissions and revive an essential ecosystem that provides many services, for people, the planet and the climate—including their vital role as a natural carbon sink. Hence, statement 1 is correct. They are mostly found in permafrost regions towards the poles and at high altitudes, in coastal areas, beneath tropical rainforest and in boreal forests. Countries with the largest peatland areas are – Russia, Canada, Indonesia, USA, Finland etc. Hence statement 2 is not correct. Recently, the Brazzaville Declaration was signed to promote better management and conservation of Cuvette Centrale Region in the Congo Basin. It was jointly signed by Democratic Republic of Congo (DRC), the Republic of Congo and Indonesia. Hence statement 3 is correct. Peatlands are important for Carbon Storage, Supporting unique and critically threatened biodiversity, supporting water cycle, supporting livelihood, as a cultural landscape and archive etc.

www.visionias.in

Join For More Update : - https://upscpdf.com

©Vision IAS

https://t.me/UPSC_PDF

https://upscpdf.com

https://t.me/UPSC_PDF

Q 12.D •



World Heritage Outlook as been developed by IUCN (International Union for Conservation of Nature). It provides the first global assessment of the conservation prospects for natural World Heritage. By assessing every natural site with World Heritage status, it recognises good conservation practice and supports the role of World Heritage sites in demonstrating excellence. It also identifies the actions needed to support sites that are facing threats, in order to improve their conservation outlook. Hence option (d) is the correct answer. IUCN is the world's oldest and largest global environmental organization. It demonstrates how biodiversity is fundamental to addressing some of the world's greatest challenges such as climate change, sustainable development and food security.

Q 13.B •



Global Forest Watch (GFW) is an online platform that provides data and tools for monitoring forests. By harnessing cutting-edge technology, GFW allows anyone to access near real-time information about where and how forests are changing around the world. It is an initiative of the World Resources Institute, with partners including Google, USAID and other academic, non-profit, public, and private organizations. Hence, option (b) is the correct answer. It is free and simple to use, enabling anyone to create custom maps, analyze forest trends, subscribe to alerts, or download data for their local area or the entire world. Users can also contribute to GFW by sharing data and stories from the ground via GFW’s crowdsourcing tools, blogs, and discussion groups. Special “apps” provide detailed information for companies that wish to reduce the risk of deforestation in their supply chains, users who want to monitor fires across Southeast Asia, and more. GFW serves a variety of users including governments, the private sector, NGOs, journalists, universities, and the general public.

Q 14.C •





5

Soil organic carbon (SOC) is the carbon associated with soil organic matter (SOM). SOM comprises the remains of plants and animals in the soil at various stages of decomposition, along with the microbial biomass and several by-products of complex biotic metabolic processes. It affects many soil properties such as hydrology, structure, and habitat. Soil organic carbon tends to be concentrated in the topsoil. The 2015 Status of the World’s Soil Resources report, released by The Food and Agriculture Organization (FAO), highlights that more carbon resides in soil than in the atmosphere and all plant life combined. Hence, statement 1 is correct. As per scientific research, more carbon resides in soil than in the atmosphere and all plant life combined; there are 2,500 billion tons of carbon in the soil, compared with 800 billion tons in the atmosphere and 560 billion tons in plant and animal life. www.visionias.in

Join For More Update : - https://upscpdf.com

©Vision IAS

https://t.me/UPSC_PDF •



https://upscpdf.com

https://t.me/UPSC_PDF

Land Degradation Neutrality (LDN) has been defined by the Parties to the UNCCD as a state whereby the amount and quality of land resources, necessary to support ecosystem functions and services and enhance food security, remains stable or increases within specified temporal and spatial scales and ecosystems. Three global indicators of LDN o Trends in land cover change (LCC): Indicating more immediate changes in land use and vegetation o Land productivity dynamics (LPD) measured as net primary productivity (NPP): Indicating relatively rapid responses of ecosystem function o Carbon stocks measured as soil organic carbon (SOC): Indicating the longer term and cumulative responses/resilience to land degradation. Hence, statement 2 is correct.

Q 15.D •









Statement 2 is not correct: Accessible India Campaign (AIC) is the nationwide flagship campaign of the Department of Empowerment of Persons with Disabilities (DEPwD), Ministry of Social Justice and Empowerment. The aim of the Campaign is to make a barrier-free and conducive environment for Divyangjans (disabled) all over the country. The campaign is based on the principles of the Social Model of Disability, that disability is caused by the way society is organised, and not the person’s limitations and impairments. The physical, social, structural and attitudinal barriers prevent People with Disabilities from participating equally in the socio-cultural and economic activities. A barrier-free environment facilitates equal participation in all the activities and promotes an independent and dignified way of life. The Medical model of disability states that disability is caused by the health condition a person has and the nature of this condition will determine what they can and can’t do. The medical model would say that in order for everyone to participate fully in society, everyone would need a non-disabled body and mind. This makes ‘disability’ the result of the person being different, not of society. Statement 1 is not correct: For creating universal accessibility for Persons with Disabilities, the campaign has been divided into three verticals: o Built Environment Accessibility ▪ An accessible physical environment benefits everyone, not just persons with disabilities. Measures should be undertaken to eliminate obstacles and barriers to indoor and outdoor facilities including schools, medical facilities, and workplaces. These would include not only buildings, but also footpaths, curb cuts, and obstacles that block the flow of pedestrian traffic. o Transportation System Accessibility ▪ Transportation is a vital component for independent living, and like others in society, PwDs rely on transportation facilities to move from one place to another. The term transportation covers a number of areas including air travel, buses, taxis, and trains. o Information and Communication Eco-System Accessibility ▪ Access to information creates opportunities for everyone in society. Access to information refers to all information. People use information in many forms to make decisions about their daily lives. This can range from actions such as being able to read price tags, to physically enter a hall, to participate in an event, to read a pamphlet with healthcare information, to understand a train timetable, or to view webpages. The Campaign was launched with some fixed targets that are yet to be achieved. While presenting the status of the campaign in Lok Sabha, the government has stated that due to ‘slow progress’ the deadline has been extended to March 2020.

Q 16.A • •





6

To mark its 75th anniversary in 2020, the United Nations has launched the biggest-ever global conversation on the role of global cooperation in building the future. The UN75 initiative itself is about the future, and how we can shape it, together. It is about dialogue – about the UN listening and learning, responding and engaging with as many different people and constituencies as possible. Just when we need collective action more than ever, support for global cooperation is flagging. In many countries, public trust in traditional institutions is in decline and relations between countries are under strain. Dialogue – and action – on global issues could not be more urgent. Through these conversations, the UN aims to build a global vision of 2045 – its centenary, increase understanding of the threats to that future, and support enhanced international cooperation to realise that vision. – UN75 The UN Environment Programme is supporting this effort by providing scientific and technical expertise on issues related to its mandate including the climate crisis, biodiversity loss, pollution and the environmental dimensions of the Sustainable Development Goals. Hence statement (a) is the correct answer. www.visionias.in

Join For More Update : - https://upscpdf.com

©Vision IAS

https://t.me/UPSC_PDF

https://upscpdf.com

https://t.me/UPSC_PDF

Q 17.C •



The United States and China have reached a historic and enforceable agreement on a Phase One trade deal that requires structural reforms and other changes to China’s economic and trade regime in the areas of intellectual property, technology transfer, agriculture, financial services, and currency and foreign exchange. The Phase One agreement also includes a commitment by China that it will make substantial additional purchases of the United States goods and services in the coming years. Importantly, the agreement establishes a strong dispute resolution system that ensures prompt and effective implementation and enforcement.

Q 18.B

Q 19.C •



• •

7

Aichi Targets (under CBD) are biodiversity protection targets to be achieved by 2020. This short-term plan is officially known as the “Strategic Plan for Biodiversity 2011-2020. India has declared twelve national biodiversity targets corresponding to the twenty AichiBiodiversity Targets. Hence statement 1 is correct. Recently the Government of India released a report titled 'Implementation of India’s National Biodiversity Action Plan: An Overview 2019'. It highlights India's action and performance with respect to the Aichi Biodiversity Targets which are to be achieved by the year 2020. The NBT 9 was achieved by 2014 to achieve ABT 16 by ratifying Nagoya Protocol on Access and Benefit Sharing India has also achieved NBT6 which implements ABT 11 by determining area-based conservation target keeping in view the country’s national priorities aims at bringing over 20% of the geographical area of the country comprising terrestrial inland water, coastal and marine zones, areas important for species, biodiversity and ecosystem services under area-based conservation. India has already achieved its target and contributes significantly to achieving the global target. Hence statement 2 is correct. www.visionias.in

Join For More Update : - https://upscpdf.com

©Vision IAS

https://t.me/UPSC_PDF

https://upscpdf.com

https://t.me/UPSC_PDF

Q 20.C •

• •



Marine protected areas are essential to safeguard biodiversity and to sustain vibrant seas and can increase biomass and biodiversity in tropical and temperate ecosystems, as well as serve as insurance policies against the impacts of fishing and other destructive activities. All MPAs are designated for the purpose of conservation of biodiversity or cultural heritage. It is one of the most effective means of protecting marine and coastal biodiversity. The designation of MPAs and MPA networks is driven by a range of international, regional, and national obligations and initiatives. Hence statement 1 is correct. In India, a network of MPAs was established under the Wildlife Protection Act, 1972. Hence statement 2 is correct. It includes marine parks, nature reserves and locally managed marine areas that protect reefs, seagrass beds, shipwrecks, archaeological sites, tidal lagoons, mudflats, salt marshes, mangroves, rock platforms, underwater areas on the coast and the seabed in deep water. Marine Protected Areas cover many different types of protection. Some are “no-take zones or protected zones” that are essential to enable fish stocks to recover while others allow multiple uses of their resources. MPAs protect key ecosystems such as coral reefs. Not only do they act as safe breeding grounds for fish, they also generate tourism, which in turn brings jobs. Creating more Community Managed MPAs would enhance the flow of benefits to local people.

Q 21.A •





8

Statement 1 is correct: As per the Ministry of Environment, Forests and Climate Change (MoEFCC), ESZ are formed to serve as “shock absorber” for protected areas. The basic aim is to regulate certain activities around national parks and wildlife sanctuaries to minimize the negative impact on the fragile ecosystem around. Statement 2 is not correct: The relevant guidelines for identification of ESZs are released by MOEFCC, which includes suggestions from states. The demarcation around the protected area is done by the states. But the final declaration is done by MOEFCC. In 2018, Supreme Court directed that an area of 10 kilometres around 21 national parks and wildlife sanctuaries be declared as ESZs by the Ministry of Environment, Forest and Climate Change (MoEFCC) after 9 states had not carried out the required demarcation after 12 years of an earlier SC judgement. Statement 3 is not correct: As per MoEFCC guidelines broad-based thematic activities have been classified as prohibited, restricted with safeguards and permissible. Activities like commercial mining, setting of sawmills and industries causing pollution etc. are prohibited. Felling of trees, drastic change in agriculture systems, setting up of hotels and resorts etc. are regulated. Activities permitted in the areas www.visionias.in

Join For More Update : - https://upscpdf.com

©Vision IAS

https://t.me/UPSC_PDF

https://upscpdf.com

https://t.me/UPSC_PDF

include ongoing agriculture and horticulture practices by local communities, rainwater harvesting etc. Recently the Jammu and Kashmir government set up a ten-member committee to declare Srinagar’s famous Dal Lake and its surrounding areas as an eco-sensitive zone (ESZ). Q 22.B •

• •

Mattu Gulla is a variety of green brinjal grown in and around the village of Matti (also called Mattu) in Udupi, India. It was given the Geographical Indication tag in 2011. Mattu Gulla is a widely used vegetable in Udupi cuisine especially the sambar. Mattu Gulla obtained the Geographical Indication tag in 2011 for its unusual and unique taste and exclusive location of production. It is known for its unique taste and grown in Mattu, Kaipunjal and Uliyargoli villages of Udupi district. About 200 farmers cultivate this rare variety of brinjal on about 120 acres of land from October to June. This vegetable has a special relationship with the biennial Paryaya festival held in Udupi as the farmers offer the crop to the 800-year-old Sri Krishna Mutt/Temple for the festival. A dish is prepared using it and served to devotees.

Q 23.D •



Wildlife Protection Act (1972) provides for the protection of wild animals, birds and plants and matters connected with them, with a view to ensure the ecological and environmental security of India. It has six schedules which give varying degrees of protection o Species listed in Schedule I and part II of Schedule II get absolute protection — offences under these are prescribed the highest penalties o Species listed in Schedule III and Schedule IV are also protected, but the penalties are much lower o Schedule V includes the animals which may be hunted o The plants in Schedule VI are prohibited from cultivation and planting. Hence, statement 1 is not correct. Power to alter entries lies with Central Government. The Central Government may, if it is of opinion that it is expedient so to do, can add or delete any entry to or from any Schedule or transfer any entry from one Part of a Schedule to another Part of the same Schedule or from one Schedule to another, by notification. Hence, statement 2 is not correct.

Q 24.A •





Statement 1 is correct: As per Article 3 of the Ramsar Convention: Each Contracting Party shall arrange to be informed at the earliest possible time if the ecological character of any wetland in its territory and included in the List has changed, is changing or is likely to change as the result of technological developments, pollution or other human interference. According to Article 4, each Contracting Party shall promote the conservation of wetlands and waterfowl by establishing nature reserves on wetlands, whether they are included in the List or not, and provide adequately for their wardening. Statement 2 is not correct: According to The Wetlands(Conservation and Management) Rules, 2010, a "wetland" means an area or of marsh, fen, peatland or water; natural or artificial, permanent or temporary, with water that is static or flowing, fresh, brackish or salt, including areas of marine water, the depth of which at low tide does not exceed six metres and includes all inland waters such as lakes, reservoir, tanks, backwaters, lagoon, creeks, estuaries and manmade wetland and the zone of direct influence on wetlands that is to say the drainage area or catchment region of the wetlands as determined by the authority but does not include main river channels, paddy fields and the coastal wetland covered under the notification of the Government of India in the Ministry of Environment and Forest.

Q 25.D •



9

The Non-state Actor Zone for Climate Action (NAZCA) is an online portal hosted by the United Nations Framework Convention on Climate Change (UNFCCC). It highlights actions that cities, companies, investors, and regions – non-state actors – are taking to address climate change. Showing non-state climate ambition provides momentum to the climate talks and helps give governments the confidence to sign ambitious agreements at the Paris climate negotiations (COP 21). The portal covers the Parties of the (UN Climate Change) Convention. Currently, the EU is represented in the portal by its members. Announced initiatives represent initiatives announced at major climate action events and summits that have not yet been reported through one of the portal's data providers. The portal helps provide transparency and accountability for new initiatives and facilitates tracking their progress and impact. The first major step in this regard is the requirement that www.visionias.in

Join For More Update : - https://upscpdf.com

©Vision IAS

https://t.me/UPSC_PDF



https://upscpdf.com

https://t.me/UPSC_PDF

announcements are registered through one of the portal's data providers within a fixed timeframe. Announcements are highlighted across the web site through grey shading. Hence all the three statements are correct.

Q 26.C •





Led by El Salvador and together with over 70countries, the United Nations General Assembly (UNGA) proclaimed 2021–2030 as the Decade on ecosystem Restoration on March 1, 2019. In its resolution, the UNGA recalls the United Nations Environment Assembly’s resolution calling for the conservation and restoration of all ecosystems. The UN Environment Programme (UNEP) and Food and Agriculture Organization (FAO) are tasked to lead the implementation. The UN Decade on Ecosystem Restoration also aligns with the Decade of Action for the Sustainable Development Goals. The Decade, as mentioned above, is designed to achieve existing targets defined by the sustainable development Goals, the Convention on biological diversity, the UN Convention to combat desertification and other global frameworks. Hence both the statements are correct.

Q 27.B • Critically endangered is the highest risk category assigned by the IUCN (International Union for Conservation of Nature) Red List of wild species. There are five quantitative criteria to determine whether a taxon is threatened. A taxon is critically endangered when the best available evidence indicates that it meets any of the following criteria: o Populations have declined or will decrease, by greater than 80% over the last 10 years or three generations. o Have a restricted geographical range. o Small population size of less than 250 individuals and continuing decline at 25% in 3 years or one generation. o Very small or restricted population of fewer than 50 mature individuals. o High probability of extinction in the wild. Q 28.C •



• •

The World Meteorological Organization (WMO) convened the High Mountain Summit from 29 to 31 October 2019 at its headquarters in Geneva, Switzerland. The main outcome of the Summit was a call for action and a roadmap to science-based, user-driven knowledge and information systems supporting sustainable development and risk reduction in mountain and downstream regions. The Summit aims to identify priority actions on policy, science, observations, and services, addressing the effects of climate changes observed in high mountain cryosphere, and their impacts on downstream ecosystems, communities, and water resources. The World Meteorological Organization (WMO), the UN system’s authoritative global voice on weather, climate, and water, is taking the lead in addressing these challenges in collaboration with partners. Hence statement (c) is the correct answer.

Q 29.A •





10

India and the US have concluded the Industrial Security Annex agreement, which will facilitate collaboration between defence industries by supporting the secure transfer of key information and technology. Hence statements 1 is correct and statement 2 is not correct. The ISA is a part of the General Security of Military Information Agreement (GSOMIA) which was signed by India and U.S. in 2002. The ‘2+2’ dialogue is a meeting between the India Ministers for External Affairs and Defence, and the US Secretaries of State and Defence to focus on “strengthening strategic, security and defence cooperation” between the two countries. Other agreements signed: o Finalisation of Defense Technology and Trade Initiative (DTTI) Standard Operating Procedure (SOP) which will harmonise the two side’s processes for identification, development and execution of projects under the DTTI. o Tiger Triumph Exercise: It has been decided to hold the India-U.S. joint tri-services and amphibious exercise ‘Tiger Triumph’ on an annual basis. It was first held in November 2019 as a Humanitarian Assistance and Disaster Relief (HADR) Exercise. o Coalition for Disaster Resilient Infrastructure (CDRI): USA announced its commitment to being the founding member of the CDRI whose headquarters will be located in India. www.visionias.in

Join For More Update : - https://upscpdf.com

©Vision IAS

https://t.me/UPSC_PDF

https://upscpdf.com

https://t.me/UPSC_PDF

Q 30.C •





Recently, the 14th Conference of Parties (COP14) to United Nations Convention to Combat Desertification (UNCCD) was concluded in Greater Noida in India. In an unprecedented global campaign to save productive land, country parties have agreed to make the Sustainable Development Goal target of achieving land degradation neutrality by 2030 a national target for action. This year, India being the global host for COP 14 took over the COP Presidency from China for the next two years till 2021. COP 14 also adopted a landmark decision to buttress global efforts to better mitigate and manage the risks of drought and to build resilience. Attention was also drawn to the role, the private sector plays in land restoration going forward, including through promoting sustainable value chains, as well as the incentives that will draw them in, such as the regulation in support of innovation for sustainable land management and rewarding conservation, restoration and sustainable use of resources. Hence both the statements are correct.

Q 31.C • Friends of Earth International is the world’s largest grassroots environmental network, uniting 73 national member groups and some 5,000 local activist groups on every continent. Its vision is of a peaceful and sustainable world based on societies living in harmony with nature. Their mission includes: o To collectively ensure environmental and social justice, human dignity, and respect for human rights and peoples' rights so as to secure sustainable societies. o To halt and reverse environmental degradation and depletion of natural resources, nurture the earth's ecological and cultural diversity, and secure sustainable livelihoods. o To secure the empowerment of indigenous peoples, local communities, women, groups and individuals, and to ensure public participation in decision making. o To bring about transformation towards sustainability and equity between and within societies with creative approaches and solutions. o To engage in vibrant campaigns, raise awareness, mobilize people and build alliances with diverse movements, linking grassroots, national and global struggles. o To inspire one another and to harness, strengthen and complement each other's capacities, living the change we wish to see and working together in solidarity. • Presently, no Indian organization is a member of this network. • Hence both the statements are correct. Q 32.B •

• •









11

Bharat stage emission standards (BSES) are emission standards to regulate the pollutants from internal combustion engines and Spark-ignition engines equipment, including motor vehicles. The standards and the timeline for implementation are set by the Central Pollution Control Board under the Ministry of Environment, Forest and Climate Change BS-VI norms are the latest in series of emission standards which were first introduced in 2000. Statement 1 is not correct: BS-VI allows a maximum sulphur content of 10 ppm as compared to 50 ppm under BS-IV. Oxides of nitrogen (NOx), produced as a result of combustion, will be brought down by 70 per cent for diesel, and 25 per cent for petrol engines. Another significant change will be the presence of OBD (Onboard Diagnostics) and RDE (Real Driving Emission) on all vehicles, enabling real-time tracking of emissions. Statement 2 is correct: The Ministry of Road Transport & Highways has exempted the armoured and other specialised vehicles of Indian armed forces and paramilitary forces from the new stringent vehicular emission norms (BS-VI). The exemption has been granted because these vehicles operate in remote and inhospitable terrains with most challenging operational and environmental conditions. Due to security challenges and requirements of specialized operations, the development of suitable engine compliant with the above norms would require considerable time. Further, it is difficult to maintain the ideal transportation and storage conditions of fuel in these conditions. Statement 3 is not correct: International Centre for Automotive Technology (ICAT) is the premier testing and certification agency authorized by Ministry of Road Transport and Highways for providing testing and certification services to the vehicle and component manufacturers in India and abroad. ICAT released India’s first Type Approval Certificate (TAC) for Bharat Stage - VI (BS – VI) norms for the two wheeler segment. www.visionias.in

Join For More Update : - https://upscpdf.com

©Vision IAS

https://t.me/UPSC_PDF

https://upscpdf.com

https://t.me/UPSC_PDF

Q 33.A •









The three Rio Conventions—on Biodiversity, Climate Change and Desertification—derive directly from the 1992 Earth Summit. Each instrument represents a way of contributing to the sustainable development goals of Agenda 21. The three conventions are intrinsically linked, operating in the same ecosystems and addressing interdependent issues. Convention on Biological Diversity: The objectives of the CBD are the conservation of biological diversity, the sustainable use of its components, and the fair and equitable sharing of the benefits arising from commercial and other utilization of genetic resources. The agreement covers all ecosystems, species, and genetic resources. United Nations Convention to Combat Desertification: The UNCCD aims to combat desertification and mitigate the effects of drought in countries experiencing serious drought and/or desertification, particularly in Africa, through effective actions at all levels, supported by international co-operation and partnership arrangements, in the framework of an integrated approach which is consistent with Agenda 21, with a view to contributing to the achievements of sustainable development in affected areas. United Nations Framework Convention on Climate Change: The UNFCCC sets an overall framework for intergovernmental efforts to tackle the challenge posed by climate change. Its objectives are to stabilize greenhouse-gas concentrations in the atmosphere at a level that would prevent dangerous anthropogenic interference with the climate system, within a time-frame sufficient to allow ecosystems to adapt naturally to climate change; to ensure that food production is not threatened; to enable economic development to proceed in a sustainable manner. Hence all the options are correct.

Q 34.D •





The 25th UN Climate Conference – known as COP25 – took place in Madrid against a backdrop of the growing recognition of the impacts of climate change and the need to cut greenhouse gas (GHG) emissions. The 25th Conference of Parties (CoP 25) to the United Nations Framework Convention on Climate Change fell short on its primary deliverables — rules on markets and finance for ‘loss and damage’. 2020 is a critical year for climate action on two fronts. It is the deadline for emissions reduction commitments made by developed countries in 2008 and 2009 (at Cancun and Copenhagen). It is also the deadline for the $100 billion in finance to be provided to developing countries, a commitment made by developed countries in Paris which has not been done so far. The Paris Agreement requires countries to update their national targets (nationally determined contributions or NDCs) every five years. Current NDCs have us on track to warm the planet by more than 2.5°C. Several larger developing countries, China and India foremost among them, have objected to being pushed to update national ambition without developed countries delivering on past promises on emission reduction and finance. The decision titled Chile Madrid Time for Action in COP 25 recalled the $100 billion goals and recognized the urgent need to enhance finance to developing countries in line with that goal. Hence none of the outcomes mentioned above were achieved at the recently concluded COP25.

Q 35.C •





12

Statement 1 is correct: The Biological Diversity Act, 2002 is an Act of the Parliament of India for the preservation of biological diversity in India and provides a mechanism for equitable sharing of benefits arising out of the use of traditional biological resources and knowledge. The Act was enacted to meet the obligations under the Convention on Biological Diversity (CBD), to which India is a party. Statement 2 is correct: The Biodiversity Act - 2002 provides for a three-tiered structure - at the national, state and local levels. At the local level, the Biodiversity Management Committees (BMCs) are to be established by institutions of local self-government for implementation of specific provisions of the Act and Rules. At the state level, the State Biodiversity Boards (SBBs) are established to deal with all matters relating to the implementation of the Act and the Rules. At the national level, the National Biodiversity Authority (NBA) is established to deal with all matters relating to the implementation of the Act and the Rules. Each of these structures is required to be connected for decision-making processes on various issues, including on issues of access and benefit-sharing (ABS). The act provides for no requirement for seeking permission for carrying out research in biological resources if it is carried out in India by Indians, as well as under collaborative research projects that have been drawn within the overall policy guidelines formulated by the Central Government. The only situations that would require the permission of the NBA are: www.visionias.in

Join For More Update : - https://upscpdf.com

©Vision IAS

https://t.me/UPSC_PDF

https://upscpdf.com

https://t.me/UPSC_PDF

o



when the results of any research which has made use of the country's biodiversity is sought to be commercialized, o when the results of the research are shared with a foreigner or foreign institution, and o when a foreign institution/individual wants access to the country's biodiversity for undertaking research and o when any intellectual property protection is sought on Indian biological resources. The act provides for designating Biodiversity Heritage Sites (BHS). These are areas of biodiversity importance, which harbour rich biodiversity, wild relatives of crops, or areas, which lie outside the protected area network. The purpose is not to cover the already designated protected areas such as national parks and wildlife sanctuaries.

Q 36.A •





The COP is the supreme decision-making body of the Convention. All States that are Parties to the Convention are represented at the COP, at which they review the implementation of the Convention and any other legal instruments that the COP adopts and take decisions necessary to promote the effective implementation of the Convention, including institutional and administrative arrangements. The COP meets every year, unless the Parties decide otherwise. The first COP meeting was held in Berlin, Germany in March, 1995. The COP Presidency rotates among the five recognized UN regions - that is, Africa, Asia, Latin America and the Caribbean, Central and Eastern Europe and Western Europe and Others. Statement 1 is correct: The Conference of the Parties, the supreme body of the Convention, shall serve as the meeting of the Parties to the Paris Agreement. All States that are Parties to the Paris Agreement are represented at the Conference of the Parties serving as the meeting of the Parties to the Paris Agreement (CMA), while States that are not Parties participate as observers. The CMA oversees the implementation of the Paris Agreement and takes decisions to promote its effective implementation. The CMA meets annually during the same period as the COP. Parties to the Convention that are not Parties to the Paris Agreement are able to participate in the CMA as observers, but without the right to take decisions. The functions of the CMA relating to the Paris Agreement are similar to those carried out by the COP for the Convention. Statement 2 is not correct: Recently at COP25 of UNFCCC, the CMA2 was held.

Q 37.D •





• •

The Sarus crane is the tallest flying bird in the world standing 152-156 cm tall with a wingspan of 240cm. It has a predominantly grey plumage with a naked redhead and upper neck and pale red legs. Females are smaller, growing to about 35-40kg, while the males grow bigger, up to 40-45kg. It is a social creature, found mostly in pairs or small groups of three or four. Known to mate for life with a single partner, its breeding season coincides with heavy rainfall in monsoon. Nests are constructed on the water in natural wetlands or in flooded paddy fields. Usually, a clutch has only one or two eggs, which are incubated by both parents for a period of 26 to 35 days. The juveniles follow their parents from the day of birth. The Sarus crane has three disjunct populations in the Indian sub-continent, south-east Asia and northern Australia with an estimated global population of 25,000-37,000 individuals. In the Indian subcontinent, it is found in northern and central India, Terai Nepal and Pakistan. It was once a common sight in the paddy fields of Uttar Pradesh, Bihar, Rajasthan, West Bengal, Gujarat, Madhya Pradesh and Assam. Their population is now on the decline with only 15,000-20,000 found in India, a majority of which are in Uttar Pradesh. The Sarus crane is known for its ability to live in association with humans, inhabiting open, cultivated, well-watered plains, marshlands and jheels. These areas suit them well for foraging, roosting and nesting. It is listed in Schedule IV of the Wildlife (Protection) Act 1972 and as Vulnerable on IUCN Red List Hence all the statements given in the question are correct.

Q 38.C •





13

The Scheduled Tribes and Other Traditional Forest Dwellers (Recognition of Forest Rights) Act, 2006 was enacted to assert the rights of tribal communities over the forestland over which they were traditionally dependent. This Act is crucial to the rights of millions of tribals and other forest dwellers in different parts of our country as it provides for the restitution of deprived forest rights across India, including both individual rights to cultivated land in forestland and community rights over common property resources. The Act is significant as it provides scope and historic opportunity of integrating conservation and livelihood rights of the people. Following are the Rights provided under the act: www.visionias.in

Join For More Update : - https://upscpdf.com

©Vision IAS

https://t.me/UPSC_PDF • • • • •

• •

https://upscpdf.com

https://t.me/UPSC_PDF

For the first time, the Forest Rights Act recognises and secures Community Rights or rights over common property resources of the communities in addition to their individual rights. Rights in and over disputed land Rights of settlement and conversion of all forest villages, old habitation, un-surveyed villages and other villages in forests into revenue villages. Right to protect, regenerate or conserve or manage any community forest resource which the communities have been traditionally protecting and conserving for sustainable use. Right to intellectual property and traditional knowledge related to biodiversity and cultural diversity. Rights of displaced communities. Right to in situ rehabilitation including alternative land in cases where the Scheduled Tribes or other traditional forest dwellers have been illegally evicted or displaced from forest land of any description without receiving their legal entitlement to rehabilitation. Rights over developmental activities. Hence all the statements are correct.

Q 39.A •

• • •

Amidst the growing concern of need of a planetary defence mechanism, scientists are studying asteroids and trying to find ways to deflect them from a collision course with Earth. One such project is the Asteroid Impact and Deflection Assessment (AIDA), which includes NASA’s Double Asteroid Redirection Test (DART) mission and the European Space Agency’s (ESA) Hera. The European Space Agency (ESA) has approved the budget of Hera, the European component of the mission to slam a spacecraft into an asteroid. Hence, statement 2 is incorrect. The project aims to study the effectiveness of an impact to ward off an impending asteroid threat. Hence, statement 1 is correct. One such project is Didymos asteroid system: o The twin-asteroid system Didymos is a binary near-Earth asteroid. o According to NASA, while the primary body of Didymos is approximately 780 meters across, its secondary body or “moonlet” is about 160-meters in size, which is more typical of the size of asteroids that could pose the most likely significant threat to Earth. So, Didymos makes a suitable target for NASA and ESA’s mission.

Q 40.B •





The Global Snow Leopard and Ecosystem Protection Program (GSLEP) seeks to address high-mountain development issues using the conservation of the charismatic and endangered snow leopard as a flagship. It is an alliance of that unites Snow Leopard range countries ( Russia, China, Mongolia, Kazakhstan, Kyrgyzstan, Uzbekistan, Tajikistan, India, Nepal, Bhutan, Afghanistan and Pakistan.), nongovernmental and inter-governmental organizations, local communities, and the private sector around a shared vision to conserve snow leopards and their valuable high-mountain ecosystems. Hence, statement 1 is not correct. The snow leopard range countries and partners unanimously agree to the shared goal of the GSLEP for the 7 years through 2020. One of the objectives is to work together to identify and secure at least 20 snow leopard landscapes across the Snow Leopard's range by 2020 or, in shorthand – “Secure 20 by 2020". Hence, statement 2 is correct. Secure snow leopard landscapes are defined as those that contain at least 100 breeding age snow leopards conserved with the involvement of local communities, support adequate and secure prey populations, and have functional connectivity to other snow leopard landscapes, some of which cross international boundaries.

Q 41.A •

• •



14

Article 13 of the Paris Agreement established an enhanced transparency framework for action and support, with built-in flexibility which takes into account Parties’ different capacities and builds upon collective experience. Hence, option (a) is the correct answer. The purpose of the framework for transparency of support is to provide clarity on support provided and received by relevant individual Parties in the context of climate change actions. Under the enhanced transparency framework for support, developed country Parties shall, and other Parties that provide support should, provide information on financial, technology transfer and capacitybuilding support provided to developing country Parties. This information shall undergo a technical expert review. In addition, each Party shall participate in a facilitative, multilateral consideration of progress. Developing countries are also required to provide information on financial, technology transfer and capacity-building support needed and received. www.visionias.in

Join For More Update : - https://upscpdf.com

©Vision IAS

https://t.me/UPSC_PDF

https://upscpdf.com

https://t.me/UPSC_PDF

Q 42.A •









The Emissions trading is a market-based programme where the government sets a cap on emissions and allows industries to buy and sell permits to stay below the cap. Gujarat has become the first to launch emissions trading for Particulate Matter (PM) globally. Hence, option (a) is the correct answer. Being initiated in Surat by the Gujarat Pollution Control Board (GPCB), the emission trading scheme (ETS) was designed with the help of a team of researchers from the Energy Policy Institute at the University of Chicago (EPIC), the Economic Growth Center at Yale University and others from The Abdul Latif Jameel Poverty Action Lab (J-PAL). Under this system, the regulator first defines the total mass of pollution that can be put into the air over a defined period by all factories put together. Then, a set of permits is created, each of which allows a certain amount of pollution, and the total is equal to the cap. These permits are the quantity that is bought and sold. Each factory is allocated a share of these permits (this could be equal or based on size or some other rule). After this, plants can trade permits with each other. GPCB chose Surat as it is a densely-populated industrial centre where textile and dye mills cause a significant amount of air pollution. It will be implemented across 350 industrial units in the city in a phased manner, with around 160 units in the first phase.

Q 43.C •

• • • •

The Reserve Bank of India decided to conduct its version of ‘Operation Twist’ through simultaneous purchase and sale of government securities under Open Market Operations (OMOs) for Rs 10,000 crore each Operation Twist is the name given to a US Federal Reserve monetary policy operation. It involves the purchase and sale of government securities to boost the economy by bringing down long-term interest rates. Hence option (c) is the correct answer. It normally leads to lower longer-term yields, which will help boost the economy by making loans less expensive for those looking to buy homes, cars and finance projects. But saving becomes less desirable because it doesn’t pay as much interest.

Q 44.C •







Committee for Purpose of Control & Supervision of Experiments on Animals (CPCSEA) is a statutory body under Section 15 of Prevention of Cruelty to Animals Act, 1960 with headquarters at Chennai. The mandate of this Committee is to ensure that while conducting various types of experiments, in connection with medical research or education, animals are not subjected to avoidable pain or suffering. For this purpose, under the delegated powers, the Committee formulated the ‘Breeding of and Experiments on Animals (Control and Supervision) Rules, 1998’ which were amended in 2001 and then in 2006, to regulate the experimentation on animals. The main functions of CPCSEA are: o Registration of establishments conducting animal experimentation or breeding of animals for this purpose. o Selection and assignment of nominees for the Institutional Animal Ethics Committees of the registered establishments. o Approval of Animal House Facilities on the basis of reports of inspections conducted by CPCSEA. o Permission for conducting experiments involving use of animals. o Recommendation for import of animals for use in experiments. o Action against establishments in case of established violation of any legal norm/stipulation. o Conduct of Training Programmes for the Nominees of CPCSEA. o Conduct/Support of Conference/workshop on Animal Ethics. Hence both the statements are correct.

Q 45.C •



15

Recently, the 2019 Refinement to the 2006 IPCC Guidelines for National Greenhouse Gas Inventories was adopted and accepted during the 49th Session of the IPCC in May 2019. It was prepared by the Task Force on National Greenhouse Gas Inventories (TFI). The 2019 Refinement was considered in May 2019 during the IPCC’s 49th Session (Kyoto, Japan) and adopted/accepted on 12 May 2019. About the Task Force on National Greenhouse Gas Inventories (TFI): The TFI develops and refines an internationally-agreed methodology and software for the calculation and reporting of national www.visionias.in

Join For More Update : - https://upscpdf.com

©Vision IAS

https://t.me/UPSC_PDF





https://upscpdf.com

https://t.me/UPSC_PDF

GHG emissions and removals and encourages the use of this methodology by countries participating in the IPCC and by signatories of the United Nations Framework Convention on Climate Change (UNFCCC). The TFI was established by the IPCC, at its 14th session (October 1998), to oversee the IPCC National Greenhouse Gas Inventories Programme (IPCC-NGGIP). The objectives of the TFI are: o to develop and refine an internationally-agreed methodology and software for the calculation and reporting of national GHG emissions and removals; and o to encourage the widespread use of this methodology by countries participating in the IPCC and by signatories of the United Nations Framework Convention on Climate Change (UNFCCC). Hence both the statements are correct.

Q 46.C •







The ‘Nagpur Resolution - A holistic approach for empowering citizens’ was adopted during the Valedictory session of the two-day Regional Conference on ‘Improving Public Service Delivery – Role of Governments’, in Nagpur, Maharashtra. The conference was organised by the Department of Administrative Reforms and Public Grievances (DARPG), Government of India, in collaboration with the Government of Maharashtra and the Maharashtra State Commission for Right to Public Services. The resolution stresses on the positive approach, transparency, corruption-free system, fast track decision making and social sensitivity as essential to good governance. The team spirit and humane approach towards the subordinates is the most important quality of the successful leaders. Emphasis is given to the need to understand the spirit behind the law, independent and impartial decision making within the domain of law and development-orientated approach with sensitivity towards the last man in the queue is important for job satisfaction. The Resolution focuses on dynamic policy-making and strategic decisions, monitoring of implementation, the appointment of key personnel, coordination and evaluation, and achieving a sense of common identity by the exchange of technical expertise in the areas of Improved Service Delivery between paired States under the Ek Bharat Shreshtha Bharat Program.

Q 47.B •





Scientist have spotted a new species of signal fish, the first one of genus recorded in Indian waters. Although more than 2450 species of marine fishes have been discovered in India so far, this is the first time they have spotted the presence of a signal fish, off the Kerala coast. Scientists from the University of Kerala and the Ocean Science Foundation, USA have reported the discovery of the new species of signal fish, the first one of the genus recorded in Indian water. The signal fishes are spotted in the coral reefs. the newly identified signal fish, Pteropsaron indicum, also has bold yellow markings on its body to grab the attention of its fellow mates. The species are typically tiny and fragile (most less than 50 mm SL, and some less than 30 mm SL) and usually found below normal diving depths, more than 50 m deep. So it indicates the presence of patchy corals off the Kerala coast The discovered species is the world’s largest signal fish and the unexplored features and rarity of this species opens up new avenues of research in marine biodiversity in India.

Q 48.C •







16

Biopiracy is a situation where indigenous knowledge of nature, originating with indigenous people, is used by others for profit, without permission from and with little or no compensation or recognition to the indigenous people themselves. It seeks to establish exclusive monopoly control (patents or intellectual property) over these resources and knowledge. Developed countries are exploiting developing countries genetic resources and indigenous communities traditional knowledge in the name of patents on the inventions derived from those genetic resources. Hence statement 1 is correct. Example: use of indigenous knowledge of medicinal plants for patenting by medical companies without recognizing the fact that the knowledge is not new, or invented by the patentee. Biomining is the process of using microorganisms (microbes) to extract metals of economic interest from rock ores or mine waste. Biomining techniques may also be used to clean up sites that have been polluted with metals. Bioprospecting, also known as biodiversity prospecting, is the exploration of biological material for commercially valuable genetic and biochemical properties. In simple terms, this means the investigation of living things to see how they can be commercially useful to humans. Small samples of natural www.visionias.in

Join For More Update : - https://upscpdf.com

©Vision IAS

https://t.me/UPSC_PDF

https://upscpdf.com

https://t.me/UPSC_PDF

resources are collected for their potential value to the industry, particularly in the pharmaceutical, biotechnology and agri-business fields. Local communities close to where the material originates may have specialised knowledge on how the resources are used, which can also be collected, and this is known as traditional or indigenous knowledge (IK). Hence statement 2 is correct. Q 49.B • 3 and 4 are not part of Trans Himalaya. • There are 10 Biogeographic Zones in India and 26 Biogeographic Provinces.

Q 50.B •



The National Centre for Sustainable Coastal Management (NCSCM), Chennai is a research institute under the Ministry of Environment, Forest and Climate Change (MoEF&CC), Government of India with a vision to manage the Indian coast in a sustainable manner. It was established in the year 2011. Hence statement 1 is not correct. It delineates and maps the Ecologically Sensitive Areas and Critically Vulnerable Coastal Areas along the coast of India. Hence statement 2 is correct.

Q 51.A • • •

The Union Home Ministry has alerted the states, warning them about the vulnerability of the Android operating system to a bug called 'StrandHogg'. It is a vulnerability in android which allows real-time malware applications to pose as genuine applications and access user data of all kinds. It can listen to the conversations, access photo album, read/send messages, make calls, record conversations, get login credentials to various accounts, access private images, files, contact details, call logs and location information without being apparent to the affected users.

Q 52.A • The Global Environment Facility (GEF) was established on the eve of the 1992 Rio Earth Summit to help tackle most pressing environmental problems. The GEF unites countries in partnership with international institutions, civil society organizations (CSOs), and the private sector to address global environmental issues while supporting national sustainable development initiatives. The following funds are managed by GEF: o Special Climate Change Fund. Hence option 1 is correct. o Least Developed Countries Fund. Hence option 2 is correct. 17

www.visionias.in

Join For More Update : - https://upscpdf.com

©Vision IAS

https://t.me/UPSC_PDF

https://upscpdf.com

https://t.me/UPSC_PDF

o



GEF Trust Fund - Climate Change focal area Adaptation Fund (AF) o It was established in 2001 to finance concrete adaptation projects and programmes in developing country Parties to the Kyoto Protocol that are particularly vulnerable to the adverse effects of climate change. The Adaptation Fund is financed with a share of proceeds from the clean development mechanism (CDM) project activities and other sources of funding. o It is supervised and managed by the Adaptation Fund Board (AFB). The AFB is composed of 16 members and 16 alternates and meets at least twice a year (Membership of the AFB). Hence option 3 is not correct.

Q 53.D • •

• • • • •

• • •

The GCC is a political and economic alliance of countries in the Arabian Peninsula. OIC (Organisation of Islamic Cooperation) states that it is the collective voice of the Muslim world and works to safeguard and protect the interests of the Muslim world in the spirit of promoting international peace and harmony. Hence statement 1 is not correct. Its headquarters is located at Riyadh, Saudi Arabia. It was established in 1981 to foster socio-economic, security, and cultural cooperation. Bahrain, Kuwait, Oman, Qatar, Saudi Arabia, and the UAE are its members. They gather every year to discuss cooperation and regional affairs. Saudi Arabia, Bahrain, the United Arab Emirates (UAE) and Egypt have imposed an air, land and sea blockade on Qatar since June 2017. The three countries decided they could no longer tolerate Qatar’s support of the Muslim Brotherhood and what they see as Qatar’s interference in the internal affairs of the other GCC member states. Resulting in severed transport links, travel restrictions for Qatari citizens and residents, and various other sanctions. The blockading quartet has accused Qatar of supporting "terrorism". Qatar is still a member of the GCC. Hence statement 2 is not correct.

About OIC • •

It was established in 1969 with HQ at Jeddah, Saudi Arabia It is the second-largest inter-governmental organization after the United Nations with a membership of 57 states spread over four continents.

Q 54.C •



The Global Restoration Council is a voluntary, non-departmental entity supported by the World Resources Institute on behalf of the Global Partnership for Forest Landscape Restoration (GPFLR) and in support of other organizations that are actively engaged in restoration activities. It aims to harness the collective wisdom, influence and energy of its members to catalyze and sustain a global movement for restoration. Hence statement 1 is correct. The Global Restoration Initiative works with governments and international partners to inspire, enable and implement restoration on degraded landscapes, returning them to economic and environmental productivity. Alongside IUCN and other partners, WRI has identified more than two billion hectares of cleared and degraded forest and agricultural lands suitable for restoration – an area roughly twice the size of China. Using this data as a foundation, we work to promote restoration of degraded lands back into natural forests, agroforestry systems, or productive agriculture. Embracing forest and landscape restoration will allow for a world in which the amount of forest cover grows while the productivity of existing agricultural land increases. Hence statement 2 is correct.

Q 55.A •



18

Statement 1 is correct: GreenCo Rating is the "first of its kind in the World" holistic framework that evaluates companies on the environmental friendliness of their activities using a life cycle approach. Implementation of GreenCo rating provides leadership and guidance to companies on how to make products, services and operations greener. Industry personnel are trained on the latest Green concepts and facilitated for implementing better systems and implementing global best practices in green. The life cycle approach involves an emphasis on product design, materials used, procurement, vendor management, logistics, packaging, manufacturing, distribution, product use, disposal and recycling. Statement 3 is not correct: It has been developed by the Confederation of Indian Industry.

www.visionias.in

Join For More Update : - https://upscpdf.com

©Vision IAS

https://t.me/UPSC_PDF •

https://upscpdf.com

https://t.me/UPSC_PDF

Statement 2 is correct: GreenCo rating is applicable to both manufacturing facilities and service sector units. The rating is implemented at the unit or facility level. The unit or facility has to be in operation for a minimum period of 3 years. In the case of new plants/ facilities minimum, 2 years operation is required.

Q 56.D •



The Nagoya Protocol on Access to Genetic Resources and the Fair and Equitable Sharing of Benefits Arising from their Utilization (ABS) to the Convention on Biological Diversity is a supplementary agreement to the Convention on Biological Diversity. It provides a transparent legal framework for the effective implementation of one of the three objectives of the CBD: the fair and equitable sharing of benefits arising out of the utilization of genetic resources. The Access and Benefit-sharing Clearing-House (ABS Clearing-House) is a platform for exchanging information on access and benefit-sharing established by the Protocol, as part of the Clearing-House of the CBD.

Q 57.A •







Statement 1 is correct and statement 2 is not correct: Climate Change Action Programme (CCAP) is a scheme (an initiative of the Ministry of Environment, Forest and Climate Change) which was approved by the Cabinet in January 2014. Its objective is to create and strengthen the scientific and analytical capacity for assessment of climate change in the country, putting in place appropriate institutional framework for scientific and policy initiatives and implementation of climate change-related actions in the context of sustainable development. Some of the components of the CCAP scheme include the National Carbonaceous Aerosols Programme (NCAP), Long Term Ecological Observatories (LTEO), and Coordinated Studies on Climate Change for North East Region (CSCCNER). NCAP is a multi-institutional programme developed with a view to monitor and study carbonaceous aerosols including black carbon, and has three components viz., o Monitoring and assessing impacts of black carbon on snow undertaken by Ministry of Science and Technology (MoST) o Observational activities relating to aerosol and black carbon by Ministry of Earth Sciences (MoES) and Indian Space Research Organization (ISRO) o Inventory and modelling of carbonaceous aerosols including black carbon being undertaken by MoEFCC, which also coordinates overall activities under NCAP. LTEO is another project under the CCAP scheme which aims at creating a network of field sites at various ecological regions in the country viz. Western Himalayas, Eastern Himalayas, North-Western Arid Zone, Central Indian Forests, Western Ghats, Andaman & Nicobar Islands, Jammu & Kashmir, and Sundarbans, for undertaking long term observations of the effects of climate change. These field sites will be used by several institutions and scientists with expertise in various disciplines to identify patterns and drivers of change in the natural ecosystems.

Q 58.B • •

• •

The International Union for Conservation of Nature’s (IUCN) 10th Asian Elephant Specialist Group (AsESG) Meeting was held recently at Kota Kinabalu in Sabah, Malaysia. Statement 1 is not correct: The IUCN Asian Elephant Specialist Group (AsESG) is a global network of specialists (both scientists and non-scientists) concerned with the study, monitoring, management, and conservation of Asian Elephants (Elephas maximus). AsESG is an integral part of the Species Survival Commission (SSC) of the International Union for Conservation of Nature (IUCN). It aims to promote the long-term conservation of Asia’s elephants and, where possible, the recovery of their populations to viable levels. Statement 2 is correct: It shall provide the best available scientifically grounded evidence to the abundance, distribution, and demographic status of Asian elephant populations in all 13 range states. This is documented in 'Gajah', a bi-annual journal of the IUCN/SSC Asian Elephant Specialist Group (AsESG).

Q 59.D •

19

Recently, the National Tiger Conservation Authority (NTCA) has reported to the Supreme Court that African cheetahs, to be translocated in India from Namibia, will be kept at Nauradehi wildlife sanctuary in Madhya Pradesh. When Iran, the only country in the world to have a small population of Asiatic Cheetahs refused to send any to India, it was decided to introduce African Cheetahs to India instead. Hence statement 1 is not correct. www.visionias.in

Join For More Update : - https://upscpdf.com

©Vision IAS

https://t.me/UPSC_PDF •



https://upscpdf.com

https://t.me/UPSC_PDF

In 2009, Project Cheetah was launched and Kuno Wildlife Sanctuary (MP) and Shahgarh area in Rajasthan were identified as other two sites for cheetah reintroduction plan. Nauradehi was found to be the most suitable area for the cheetahs as its forests are not very dense to restrict the fast movement of Cheetah. Hence statement 2 is not correct. This move will make India the only country in the world to host six of the world's eight large cats, including lions, tigers, jaguars, panthers and leopards. Cheetah is the flagship species of the grasslands. This will help dryland ecosystems of India to return to their natural state.

Q 60.A •



• •



Most vocational training programmes focus on mechanical/technical skills rather than ‘soft’ or ‘green’ skills. Green skills contribute to preserving or restoring environmental quality for sustainable future and include jobs that protect ecosystems and biodiversity, reduce energy and minimize waste and pollution. The programme endeavours to develop green skilled workers having technical knowledge and commitment to sustainable development. It will help in skill development in the environment and forest sector to enable India's youth to get gainful employment and/or self-employment which will lead to attainment of the Nationally Determined Contributions (NDCs), Sustainable Development Goals (SDGs), National Biodiversity Targets (NBTs), as well as Waste Management Rules (2016). Hence, statement 1 is correct. The Green Skill Development Programme (GSDP) has been conceptualised and developed in MoEF&CC in consultation with the National Skill Development Agency. Hence, statement 2 is not correct. Realizing the need for developing the green skills, the Ministry of Environment, Forest & Climate Change (MoEF&CC) is utilising the vast network and expertise of Environmental Information System (ENVIS) hubs and Resource Partners (RPs). The first GSDP course was formulated for skilling Biodiversity Conservationists (Basic Course) and Parataxonomists (Advance Course) of 3 months duration each, on a pilot basis in ten select districts of the country (covering nine bio-geographic regions). BSI and ZSI were the nodal Centres for the pilot programme.

Q 61.C The NGT has the power to hear all civil cases relating to environmental issues and questions that are linked to the implementation of laws listed in Schedule I of the NGT Act. These include the following: • • • • • • •

The Water (Prevention and Control of Pollution) Act, 1974; The Water (Prevention and Control of Pollution) Cess Act, 1977; The Forest (Conservation) Act, 1980; The Air (Prevention and Control of Pollution) Act, 1981; The Environment (Protection) Act, 1986; The Public Liability Insurance Act, 1991; The Biological Diversity Act, 2002.

Importantly, the NGT has not been vested with powers to hear any matter relating to the Wildlife (Protection) Act, 1972 and Forest Rights Act 2006 and various laws enacted by States relating to forests, tree preservation etc. Q 62.B •





The Green Climate Fund (GCF) is a global fund created to support the efforts of developing countries to respond to the challenge of climate change by limiting or reducing their greenhouse gas (GHG) emissions and adapt to climate change. It seeks to promote a paradigm shift to low-emission and climate-resilient development by taking into account the needs of nations that are particularly vulnerable to climate change impacts. It was set up by the 194 countries who are parties to the United Nations Framework Convention on Climate Change (UNFCCC) in 2010, as part of the Convention’s financial mechanism. It aims to deliver equal amounts of funding to mitigation and adaptation, while being guided by the Convention’s principles and provisions. It will support a UNDP project in India for climate resilient livelihood for coastal areas of Andhra Pradesh, Maharashtra and Odisha. Hence option (b) is the correct answer.

Q 63.B •

20

The Bureau was constituted by amendment to the Wild Life (Protection) Act, 1972. It has been envisaged as a multidisciplinary statutory body that will have officials from forests, police, customs and other www.visionias.in ©Vision IAS

Join For More Update : - https://upscpdf.com

https://t.me/UPSC_PDF

• •



https://upscpdf.com

https://t.me/UPSC_PDF

similar agencies. The constitution is specified in Section 38 (Y) of the Wild Life (Protection) Act, 1972. Hence, statement 1 is not correct. It is a statutory body under the Ministry of Environment Forest and Climate Change to combat organized wildlife crime in the country and was constituted in 2007. The Wildlife Crime Control Bureau was created to complement the existing state machinery to deal with the wildlife crime having ramifications beyond state and national borders. It collects and collates intelligence related to organized wildlife crime activities and to disseminate the same to State and other enforcement agencies for immediate action so as to apprehend the criminals and to establish a centralized wildlife crime data bank. It also coordinates the actions by various officers, State Governments and other authorities in connection with the enforcement of the provisions of the Act. Hence, statement 2 is correct. It also assists and advises the Customs authorities in inspection of the consignments of flora & fauna as per the provisions of Wild Life Protection Act, CITES and EXIM Policy governing such an item.

Q 64.A •







The Wildlife Conservation Society is a non-governmental body committed to saving wildlife and wild places worldwide. It does so through science, conservation, education and the management of the world’s largest system of urban wildlife parks, led by the flagship Bronx Zoo. It works in partnership with the government and other local organizations to deal with conservation challenges like human-wildlife interactions etc. Wildlife Institute of India (WII): This is an Autonomous Institution of the Ministry of Environment, Forest and Climate Change, Government of India. Established in 1982, Wildlife Institute of India (WII) is an internationally acclaimed Institution, which offers training program, academic courses and advisory in wildlife research and management. The Institute is actively engaged in research across the breadth of the country on biodiversity-related issues. Wildlife Trust of India (WTI) is a leading Indian nature conservation organisation committed to the service of nature. Its mission is to conserve wildlife and its habitat and to work for the welfare of individual wild animals, in partnership with communities and governments. WTI’s team of 150 dedicated professionals work towards achieving its vision of a secure natural heritage of India, in six priority landscapes, knit holistically together by nine key strategies or Big Ideas. Hence option (a) is the correct answer. Other organizations under the Ministry of Environment, Forest and Climate Change include G.B. Pant National Institute of Himalayan Environment & Sustainable Development, Indian Council of Forestry Research and Education, Indian Institute of Forest Management, Indian Plywood Industries Research and Training Institute, Salim Ali Center for Ornithology and Natural History (SACON).

Q 65.B • • • •

• •

• • •

Universal Postal Union is the specialized agency of the United Nations. Hence statement 1 is correct. It was established in 1874 and is the second oldest international organization worldwide after the International Telecommunication Union (1865). Its headquarter is located in Bern, Switzerland, and consists of 192 member countries. Hence statement 2 is not correct. The UPU consists of four bodies: o The Congress o The Council of Administration o The Postal Operations Council o The International Bureau With its 192 member countries, the UPU is the primary forum for cooperation between postal sector players. It helps to ensure a truly universal network of up-to-date products and services. The organization fulfils an advisory, mediating and liaison role, and provides technical assistance where needed. It sets the rules for international mail exchanges and makes recommendations to stimulate growth in mail, parcel and financial services volumes and improve quality of service for customers It frames rules for international mail exchange and fixes rates for international postal services. It performs advisory, mediating and liaison role, and provides technical assistance where needed. India joined the UPU in 1876. Hence statement 3 is correct.

Q 66.C •

21

“Biodiversity Heritage Sites” (BHS) are well defined areas that are unique, ecologically fragile ecosystems - terrestrial, coastal and inland waters and, marine having rich biodiversity. Under Section 37 of Biological Diversity Act, 2002 (BDA) the State Government in consultation with local bodies may www.visionias.in

Join For More Update : - https://upscpdf.com

©Vision IAS

https://t.me/UPSC_PDF





https://upscpdf.com

https://t.me/UPSC_PDF

notify in the official gazette, areas of biodiversity importance as Biodiversity Heritage Sites (BHS). Hence, both statement 1 and statement 2 are not correct. Recently, Ameenpur Lake became the first water body in the country to be declared a Biodiversity Heritage Site. It is located on the western fringes of Hyderabad in Telangana state. It is a man-made lake and was constructed during the reign of Ibrahim Qutab Shah, who ruled the kingdom of Golconda between 1550 and 1580. Hence, statement 3 is correct. The Biodiversity Management Committee or other appropriate institution as determined by relevant local body in the absence of BMCs, which in addition to their duties defined in the Act, may also take care of the management of each BHS.

Q 67.B • •

• •

Ocean Energy Systems (OES) is an intergovernmental collaboration between countries which operates under framework established by the International Energy Agency (IEA) in 2001. It seeks to harness energy from all forms of ocean renewable resources through international cooperation and information exchange by promoting advance research, development and technologies. Hence only statement 2 is correct. India has decided to become member country of the International Energy Agency – Ocean Energy Systems (IEA-OES) to boost for country’s quest for tapping ocean energy sources. Key facts: o By becoming a member India will have access to advanced Research & Development (R&D) teams and technologies across the world. o India will also become partner in developing test protocols along with other countries which will in turn help in testing Indian prototypes as per international requirements and norms. Earth System Science Organisation – National Institute of Ocean Technology (ESSO-NIOT) under the aegis of Union Ministry of Earth Sciences will be the nodal agency for the membership. o It can also help India to leverage benefits from joint cooperative programs with institutes of member countries could be taken up. Benefits By becoming member of IEA-OES, India (power deficit country) can leverage study of ocean renewable energies. o It will also help to acquire modern technologies to harness ocean energy and overcome technological challenge due vagaries of the sea ocean energy. Provide technology to India (tropical country) to harness Ocean Thermal Energy Conversion (OTEC) due to high sea surface temperatures.

Q 68.D •

• • •

• •



A Graded Response Action Plan (GRAP) is a set of stratified actions that are taken once the pollution level reaches a certain specified limit. The Supreme Court had mandated the Environmental Pollution Control Authority (EPCA) to come up with such a plan. In January 2017, Government notified Grades Response Action Plan to combat air pollution in NCR region. Based on the air quality the grades have been classified as Emergency, Severe, Very Poor and Moderate poor. It will be enforced by the Environment Pollution Control Authority (EPCA). It defines the measures to be taken based on air quality on the basis of PM 2.5 and PM 10 in the atmosphere. Under this plan emergency measures will be automatically enforced in NCR if the level of PM2.5 breaches 300 micrograms per cubic metre (µgm/m3) and PM10 levels stay above 500 (µgm/m3) for two consecutive days. The plan recommends measures like odd-even car rationing scheme and imposing a ban on construction activities to combat air pollution. It also lists a number of other measures such as closing brick kilns, stone crushers, hot mix plants and intensifying public transport services and increasing the frequency of mechanised cleaning and the sprinkling of water on roads. For enforcement of the action plan, the Union Government has assigned the task of implementation of the Graded Response Action Plan to the EPCA (Environment Pollution Control Authority) in pursuance of sub-section (1) of section 3 of the Environment (Protection) Act, 1986 (29 of 1986).

Q 69.A •

22

The Bonn Challenge is a global effort to bring 150 million hectares of the world’s deforested and degraded land into restoration by 2020, and 350 million hectares by 2030. It was launched in 2011 by the Government of Germany and IUCN, and later endorsed and extended by the New York Declaration on Forests at the 2014 UN Climate Summit. Underlying the Bonn Challenge is the forest landscape restoration (FLR) approach, which aims to restore ecological integrity at the same time as improving human well-being through multifunctional landscapes. www.visionias.in

Join For More Update : - https://upscpdf.com

©Vision IAS

https://t.me/UPSC_PDF •





https://upscpdf.com

https://t.me/UPSC_PDF

AFR100 (the African Forest Landscape Restoration Initiative) is a country-led effort to bring 100 million hectares of land in Africa into restoration by 2030. AFR100 contributes to the Bonn Challenge, the African Resilient Landscapes Initiative (ARLI), the African Union Agenda 2063, the Sustainable Development Goals and other targets. Initiative 20x20 is a country-led effort to bring 20 million hectares of land in Latin America and the Caribbean into restoration by 2020. The initiative was launched formally at COP 20 in Lima—supports the Bonn Challenge, a global commitment to bring 150 million hectares of the world’s deforested and degraded land into restoration by 2020, and 350 million hectares by 2030, and the New York Declaration on Forests that seeks to restore 350 million hectares by 2030. Hence option (a) is the correct answer.

Q 70.A •





In 2012, the Government of India initiated the India Biodiversity Awards to recognize and honour outstanding models of biodiversity conservation, sustainable use and governance at the grassroots level. It is a joint initiative by the Ministry of Environment, Forest and Climate Change, National Biodiversity Authority and the United Nations Development Programme. Hence statement 1 is correct. It is an innovative mechanism to identify and recognize the efforts of individuals, communities and institution working towards biodiversity conservation, sustainable use of biological resources, access and benefit-sharing and biodiversity governance. The winners of India Biodiversity Awards 2020 will be given a Memento, a Certificate and a cash prize of Rs. 2 lakhs each for individuals and Rs. 5 lakhs each for institutions. Hence statement 2 is not correct. They are given in four categories: o Conservation of Wild and Domesticated Species o Sustainable Use of Biological Resources o Replicable Mechanisms for Access and Benefit Sharing o Best Biodiversity Management Committees

Q 71.D •





• •

Statement 1 is not correct: Biomedical waste (BMW) is any waste produced during the diagnosis, treatment, or immunisation of humans or during animal research activities. It also includes waste generated in the production or testing of biological material or that is produced in health camps. While India has had a biomedical waste management rule since 1998, which was modified for ease in 2016, household biomedical waste has been neglected. Household biomedical waste usually gets mixed with other household waste in India, ending up in posing numerous public health hazards. Statement 2 is not correct: Recently, the Ministry of Environment, Forest and Climate Change (MoEFCC) to amend and enforce the “parent rules” of 2016, yet again, on February 19, 2019, to improve compliance and strengthen the implementation of the policy for environment viability, which are: All bedded healthcare facilities (HCFs), irrespective of their number of beds have to regularly update the BMWM register; display the monthly record on its website, based on the colour-coded scheme (Schedule I) and have to make available the annual report on its website within a period of two years from the date of publication of the BMWM (Amendment) Rules, 2018. (The rule states that: in case of all bedded health care units, maintain and update on day to day basis the bio-medical waste management register and display the monthly record on its website according to the bio-medical waste generated in terms of category and colour coding as specified in Schedule I;) Hence statement 1 is correct. HCFs which have beds less than 10 shall have to comply with the output discharge standard for liquid waste generated, latest by December 31, 2019, and Corresponding duties of Ministry of Defence officials as per Schedule III: a report needs to be submitted to the Central Pollution Control Board (CPCB) after the inspection and monitoring of HCFs, exclusive and inclusive, of Medical Inspection (MI) rooms, sick bays onboard ships or submarines, station medical centres and field hospitals in forward locations operated by the Director General (DG), and Armed Force Medical Services (AFMS). Hence statement 2 is not correct.

Q 72.A •

23

Statement 1 is not correct: Article 334 of the Constitution lays down that its provisions relating to the reservation of seats for the SCs and STs and the representation of the Anglo-Indian community by nomination in the House of the People and Legislative Assemblies of the states shall cease to have effect on the expiration of the period of 70 years from the commencement of the Constitution. www.visionias.in

Join For More Update : - https://upscpdf.com

©Vision IAS

https://t.me/UPSC_PDF •





https://upscpdf.com

https://t.me/UPSC_PDF

Under article 80 of the Constitution, the Council of States (RajyaSabha) is composed of not more than 250 members, of whom 12 are nominated by the President of India from amongst persons who have special knowledge or practical experience in respect of such matters as literature, science, art and social service. Statement 2 is correct: The Women's Reservation Bill or The Constitution (108th Amendment) Bill, 2008, is a pending bill in the Parliament of India which propose to amend the Constitution of India to reserve 33% of all seats in the Lower house of Parliament of India, the Lok Sabha, and in all state legislative assemblies for women. The seats were proposed to be reserved in rotation and would have been determined by draw of lots in such a way that a seat would be reserved only once in three consecutive general elections. The Rajya Sabha passed the bill on 9 March 2010. However, the Lok Sabha never voted on the bill. The bill is still pending as it never went to the Lok Sabha. Statement 3 is not correct: The Constitution (One Hundred and Twenty-Sixth Amendment) Bill, 2019 amends provisions related to reservation of seats for SCs and STs and that reservation is the right of these under the privileged community. The Bill seeks to extend the reservation for SCs and STs by another 10 years till January 25, 2030. The bill does not talk about the Anglo Indian reservation The Bill has been passed in both the houses of the parliament.

Q 73.A •





Pair 1 is correctly matched: Sameer App, developed by the Central Pollution Control Board provides information about Air Quality Index (AQI) levels for more than 100 cities across the country. The listed cities are colour-coded, based on their AQI levels. Other than monitoring pollution levels, the app can also be used to file or track complaints related to garbage dumping, road dust, vehicular emission or other pollution issues in a particular area. Pair 2 is not correctly matched: PARIVESH (Pro-Active and Responsive facilitation by Interactive, Virtuous and Environmental Single-window Hub) is a web-based workflow application which has been developed for online submission and monitoring of the proposals submitted by the proponents for seeking Environment, Forest, Wildlife and CRZ Clearances from Central, State and district level authorities. It automates the entire tracking of proposals which includes online submission of a new proposal, editing/updating the details of proposals and displays status of the proposals at each stage of the workflow. Pair 3 is not correctly matched: e-Green Watch is an integrated & online system for monitoring of various afforestation works being carried out using CAMPA funds. It is completely transparent, reliable and accountable.

Q 74.C •



In pursuance to Article 6 of the Convention on Biological Diversity, India within five years of ratifying the Convention, had developed a National Policy and Macrolevel Action Strategy on Biodiversity in 1999. India also enacted the Biological Diversity Act in 2002, section 36 of which empowers the Central Govt. to develop a national biodiversity action plan. The NBAP draws upon the main principle in the National Environment Policy,2006 that human beings are at the center of concerns of sustainable development and they are entitled to a healthy and productive life in harmony with nature. Hence statement 1 is correct. Its objectives are: o Strengthening and integration of in situ, on-farm and ex situ conservation. o Augmentation of the natural resource base and its sustainable utilization: Ensuring inter and intragenerational equity. o Regulation of introduction of invasive alien species and their management. Hence, statement 2 is correct. o Assessment of vulnerability, and adaptation to climate change and desertification. o Integration of biodiversity concerns in economic and social development. o Development and integration of biodiversity databases. o Strengthening implementation of policy, legislative and administrative measures for biodiversity conservation and management. o The building of national capacities for biodiversity conservation and appropriate use of new technologies. o Valuation of goods and services provided by biodiversity and use of economic instruments in decision-making processes.

Q 75.D •

24

Monitoring of Illegal Killing of Elephants ( MIKE ) Programme: Mandated by COP resolution of CITES, MIKE program started in South Asia in the year 2003 with the purpose to provide information needed for elephant range States to make appropriate management and enforcement decisions, and to www.visionias.in

Join For More Update : - https://upscpdf.com

©Vision IAS

https://t.me/UPSC_PDF





https://upscpdf.com

https://t.me/UPSC_PDF

build institutional capacity within the range States for the long-term management of their elephant populations. The main objectives of the MIKE are: o To measure levels and trends in illegal hunting of elephants; o To determine changes in these trends over time; and o To determine the factors causing or associated with such changes, and to try and assess in particular to what extent observed trends are a result of any decision taken by The Conference of the Parties to CITES. MIKE Sites in India: o Chirang Ripu (Assam) o Dhang Patki (Assam) o Eastern Dooars (WB) o Deomali (Arunachal Pradesh) o Garo Hills (Meghalaya) o Mayurbhanj (Orissa) o Mysore (Karnataka) o Nilgiri (TN) o Shivalik (Uttarakhand) o Wayanad (Kerala)

Q 76.B •









Partnership for Urban Environmental Sustainability Initiative: The PUESI Partnership envisions the promotion of sustainable development in urban areas, aiming to build a knowledge hub in order to facilitate sharing of policies and best practices for urban sustainability, to provide an interface for information exchange and communication between suppliers and users of environmentally friendly technologies and products. It encourages participation of all stakeholders on the basis of the Environmentally Sound Technology Platform, by bringing together the BRICS Environmental Working Groups, municipalities, New Development Bank and other financial institutions, the private sector, international organisations, academic institutions and the scientific community. All partners are encouraged to leverage their own advantages to contribute inputs to devising and implementing activities under the strategy. The BRICS Partnership for Urban Environmental Sustainability Initiative if implemented in a holistic manner has the potential to contribute significantly to the reduction of pollution and waste being generated. BRICS Environmentally Sound Technology (BEST) Cooperation Platform for innovations, knowledge sharing and capacity building, and Clean River of BRICS program for improvement of river ecology and combating marine litter are some other initiatives undertaken by the BRICS countries in the domain of environment. Hence option (b) is the correct answer.

Q 77.D •

• •

25

The Cabinet Committee on Economic Affairs has given its approval for creation and launch of Bharat Bond Exchange Traded Fund (ETF) to create an additional source of funding for Central Public Sector Undertakings (CPSUs) Central Public Sector Enterprises (CPSEs), Central Public Financial Institutions (CPFIs) and other Government organizations. Hence statement 2 is correct Bharat Bond ETF would be the first corporate Bond ETF in the country. Hence, statement 1 is correct Features: o Being an ETF, after the NFO (new fund offer), its units can be purchased and sold on a stock exchange. The units will be held in your demat account. o Can buy units for as little as ₹1,000 and it is slated to have an expense ratio as low as 0.0005%. o Transparent NAV (Periodic live NAV during the day) o Transparent Portfolio (Daily disclosure on website) o As of now, it will have 2 maturity series - 3 and 10 years. Each series will have a separate index of the same maturity series. o An ETF is a passive mutual fund product which replicates an index; there is no selection by a fund manager. In return for giving up the chance to outperform the index, ETFs come with lower costs. In India, debt ETFs must have at least eight issuers and no single issuer can account for a weight greater than 15%. www.visionias.in

Join For More Update : - https://upscpdf.com

©Vision IAS

https://t.me/UPSC_PDF •

• • •

https://upscpdf.com

https://t.me/UPSC_PDF

The three-year Bharat Bond ETF will hold the debt of 13 public sector companies like National Highways Authority of India (NHAI), Indian Railway Finance Corporation (IRFC) and Power Grid Corporation of India. The 10-year Bharat Bond ETF will hold the debt of 12 public sector companies such as Rural Electrification Corporation (REC), National Bank for Agriculture and Rural Development (NABARD) and Power Finance Corporation (PFC). Bond ETF would offer CPSEs, CPSUs, CPFIs and other Government organizations an additional source of meeting their borrowing requirements apart from bank financing Investors holding demat account can invest in Bharat Bond ETF. Investors who do not hold a demat account have an alternative option to invest via Bharat Bond Fund of Funds having similar maturity in line with the underlying ETF. Hence statement 3 is not correct.

Q 78.B • •

The government recently thought to use Torrefaction, a Swedish technology to reduce stubble burning which is the key contributor to the sharp decline in air quality in Delhi About Torrefaction technique: o It is a thermochemical process typically at 200-350°C in the (near) absence of oxygen, at atmospheric pressure with low particle heating rates and a reactor time of one hour. o The process causes biomass to partly decompose, creating torrefied biomass or char, also referred to as bio-coal. o Bio- coal has higher energy content per unit volume, and torrefaction followed by pelletization at the harvest sites facilitates transport over longer distances. o It also avoids problems associated with the decomposition of biomass during storage. o It helps to convert about 65% of the biomass could be converted to energy.

Q 79.D •



• •



The Ganges River Dolphins inhabits the Ganges-Brahmaputra-Meghna and Karnaphuli-Sangu river systems of Nepal, India, and Bangladesh. It is among the four freshwater dolphins found in the world – the other three are- baiji found in the Yangtze River (China), the ‘bhulan’ of the Indus (Pakistan) and the ‘boto’ of the Amazon River (Latin America). It is fluviatile (riverine) in habitat, it may also be found in brackish water. It never enters the sea. Recently a study showed that rising salinity in the Sunderbans region of India is causing a decrease in population of the Ganges River Dolphins. Hence statement 2 is correct. It is the national aquatic animal and had been granted non-human personhood status by the government in 2017. Hence statement 1 is correct. It is mentioned under Schedule I of the Wildlife Protection Act (1972) and been categorised as endangered on the Red List of Threatened Species by the IUCN. Vikramshila Gangetic Dolphin Sanctuary (VGDS) in Bihar is India's only sanctuary for the Gangetic dolphin. Hence statement 3 is correct. It is a mammal and cannot breathe in the water and must surface every 30-120 seconds. Because of the sound it produces when breathing, the animal is popularly referred to as the 'Susu'.

Q 80.C • •



26

India has successfully conducted another night trial of its indigenously developed nuclear-capable Prithvi2 missile from a mobile launcher of the Integrated Test Range (ITR) at Chandipur in Odisha coast. Prithvi-2 missile o It is a tactical surface-to-surface short-range ballistic missile (SRBM) with a strike range of 350 kilometres and is capable of carrying 500-1,000 kilograms of warheads. o The state-of-the-art missile is powered by liquid propulsion twin engines and uses an advanced inertial guidance system with manoeuvring trajectory to hit its target. o The has been developed by the Defence Research and Development Organisation (DRDO) under the Integrated Guided Missile Development Program (IGMDM). o It is deployed by Strategic Forces Command (SFC) of the armed forces. o The 9-metre-tall, single-stage liquid-fuelled ‘Prithvi’ is the first missile to have been developed by DRDO under IGMDP. o The missile has been already inducted into the armoury of defence forces in 2003. Project 75 Aims at manufacturing six Scorpene-class submarines for Indian Navy under technology transfer from Naval Group of France.

www.visionias.in

Join For More Update : - https://upscpdf.com

©Vision IAS

https://t.me/UPSC_PDF o

https://upscpdf.com

https://t.me/UPSC_PDF

Under it, submarines are being built using a modular approach in which different sections are built separately and later assembled on pontoons (or floats). They are fitted with indigenously developed Air Independent Propulsion System (AIP) technology by DRDO for extended endurance.

Q 81.B •





Representing more than USD 2 trillion in assets under management, the United Nations-convened NetZero Asset Owner Alliance is an international group of institutional investors delivering on a bold commitment to transition their investment portfolios to net-zero GHG emissions by 2050 i.e. to shift their investment portfolios away from carbon-intensive industries. Hence statement (b) is the correct answer. The members of the Alliance commit to transitioning their investment portfolios to net-zero GHG emissions by 2050 consistent with a maximum temperature rise of 1.5°C above pre-industrial temperatures, taking into account the best available scientific knowledge including the findings of the IPCC, and regularly reporting on progress, including establishing intermediate targets every five years in line with Paris Agreement Article 4.9. The Alliance will seek to work with and enhance a range of other existing initiatives including o Science-based Targets Initiative o Climate Action 100+ o The Investor Energy & Climate Action Toolkit o The Investor Agenda

Q 82.B •







The Wildlife (Protection) Act, 1972 is the most important act regarding wildlife conservation in India. It prohibits the capturing, killing, poisoning or trapping of wild animals. It has the following objectives -: prohibition of hunting; protection and management of wildlife habitats; establishment of protected areas; regulation and control of trade in parts and products derived from wildlife and management of zoos. Animal Welfare Board of India o It is a statutory advisory body on Animal Welfare Laws and promotes animal welfare in the country. It was established in 1962 under Section 4 of the Prevention of Cruelty to Animals Act, 1960. Hence option 1 is not correct. o From ensuring that animal welfare laws in the country are diligently followed, to provide grants to Animal Welfare Organizations and advising the Government of India on animal welfare issues, the Board has been the face of the animal welfare movement in the country for the last 50 years. National Tiger Conservation Authority o It is a statutory body under enabling provisions of the Wildlife (Protection) Act, 1972, as amended in 2006, for strengthening tiger conservation, as per powers and functions assigned to it under the said Act. Hence option 2 is correct. o The National Tiger Conservation Authority has been fulfilling its mandate within the ambit of the Wildlife (Protection) Act, 1972 for strengthening tiger conservation in the country by retaining an oversight through advisories/normative guidelines, based on the appraisal of tiger status, ongoing conservation initiatives and recommendations of specially constituted Committees. Central Zoo Authority o It is a statutory authority established in 1992 after amending the Wildlife (Protection) Act, 1972 by adding a separate chapter. Hence option 3 is correct. o The Authority consists of a Chairman, ten members and a Member Secretary. The main objective of this Authority is to complement and strengthen the national effort in conservation of the rich biodiversity of the country, particularly the fauna as per the National Zoo Policy, 1998. Other objectives of this Authority include - enforcing minimum standards and norms for upkeep and healthcare of animals in Indian zoos and to control mushrooming of unplanned and ill-conceived zoos.

Q 83.B •

• •

27

The System of Air Quality and Weather Forecasting And Research (SAFAR) is a national initiative introduced by the Ministry of Earth Sciences (MoES) to measure the air quality of a metropolitan city, by measuring the overall pollution level and the location-specific air quality of the city. Hence statement 1 is not correct. The system is indigenously developed by the Indian Institute of Tropical Meteorology (IITM), Pune and is operationalized by the India Meteorological Department (IMD). It has a giant true color LED display that gives out real-time air quality index on a 24x7 basis with colorcoding (along with 72 hours advance forecast). www.visionias.in

Join For More Update : - https://upscpdf.com

©Vision IAS

https://t.me/UPSC_PDF • • • • •



https://upscpdf.com

https://t.me/UPSC_PDF

The ultimate objective of the project is to increase awareness among the general public regarding the air quality in their city so that appropriate mitigation measures and systematic action can be taken up. It organizes awareness drive by educating the public (prompting self-mitigation), and It also helps the policy-makers to develop mitigation strategies keeping in mind the nation’s economic development. SAFAR is an integral part of India’s first Air Quality Early Warning System operational in Delhi. In addition to regular air quality parameters like PM2.5, PM10, Sulfur Dioxide, Ozone, Nitrogen Oxides, Carbon Monoxide, the system will also monitor the existence of Benzene, Toluene and Xylene. Hence statement 2 is correct. Besides health, SAFAR system would benefit cost savings to several other sectors like agriculture, aviation, infrastructure, disaster management skill, tourism and many others, which directly or indirectly get affected by air quality and weather

Q 84.B •







The Sustainable Development Goals (SDG) India Index 2018 was the first effort by the Government of India and perhaps anywhere globally to rank sub-national governments by computing composite index on SDGs. The index, in its second year, ranked the states and UTs based on 100 indicators for 54 targets across 16 goals. The country’s overall score of 60, according to the Sustainable Development Goals Index 2019-20 released by NITI Aayog, was three points above its 2018 standing. This was primarily due to better performance related to water and sanitation; affordable and clean energy as well as industry, innovation and infrastructure. India’s worst performance was towards SDG 2: Ending hunger by 2030, where its score has fallen to 38 in the current year from 48 in the previous year. As many as 25 states and Union territories failed to address “hunger and malnutrition”, according to the NITI Ayog index. Jharkhand led from the bottom, scoring 22 off 100, followed by Madhya Pradesh (24) and Bihar (26). India slipped four points to 50 vis-a-vis ending poverty, with 22 states unable to reduce poverty. The situation was dismal for gender parity too. Goa ranked last there, with its score plummeting 31 points to 19. Hence option (b) is the correct answer.

Q 85.D •

• • •



Recently, the Society of Biotechnology of India (SBPI) was launched by former biotechnologists and technocrats of the Union Department of Biotechnology, Ministry of Science and Technology. Hence statement 1 and 2 are not correct It is a non-profit organisation. It would promote transformation changes and approaches towards core research in modern biotechnology so that the outcome could lead to more products and technologies for economic and social gain. It would complement India's efforts towards enhancing research funding of “gap areas” in infrastructure, human resources, regulatory frameworks and converting research and development leads into applications. Members of the SBPI have experience in promoting biotechnology in areas such as BT cotton, the introduction of recombinant therapeutic proteins and vaccines and fostering international collaborations. The Genetic Engineering Appraisal Committee (GEAC) is responsible for the appraisal of proposals relating to release of genetically engineered (GE) organisms and products into the environment including experimental field trials. Hence statement 3 is not correct.

Q 86.A •



28

Statement 1 is correct: The Production Gap Report – produced by leading research organizations and the UN – is the first assessment of the gap between the targets of the Paris Agreement and countries’ planned production of coal, oil and gas. It provides a new metric for assessing the world’s current pace of fossil fuel extraction and details the steps countries can take to align fossil fuel supply with Paris Agreement goals. This new report finds that the world is on track to produce far more coal, oil and gas than is consistent with limiting warming to 1.5°C or 2°C, creating a “production gap” that makes climate goals much harder to reach. It is a report that calls for a sharpened, and long overdue, focus on fossil fuels. Statement 2 is not correct: The report has been produced by United Nations Environment Programme (UNEP) produced in collaboration with the Stockholm Environment Institute (SEI), the International Institute for Sustainable Development (IISD), the Overseas Development Institute (ODI), Climate Analytics, and the Center for International Climate Research (CICERO). www.visionias.in

Join For More Update : - https://upscpdf.com

©Vision IAS

https://t.me/UPSC_PDF •

https://upscpdf.com

https://t.me/UPSC_PDF

The report finds that current government plans will result in 39 billion tonnes of carbon dioxide (CO2) emissions, or 53% more fossil fuels in 2030 than would be consistent with limiting warming to 2°C, and 120% more than would be consistent with a 1.5°C pathway. The production gap is the largest for coal, with countries planning to produce 5.2 billion tonnes, or 150% more coal, than would be consistent with a 2°C pathway and 280% more than would be consistent with a 1.5°C pathway.

Q 87.D • •

• • •





Recently review petitions were filed after the Supreme Court's pronunciation of the Ayodhya Verdict. All the review petitions were dismissed by the Supreme Court. Review petition: Under Article 137 of the Indian Constitution, the Supreme Court has the power to review any of its judgments or orders. This is termed as a review petition. The court has the power to review its rulings to correct a “patent error” and not “minor mistakes of inconsequential import”. In a 1975 ruling, SC said a review can be accepted “only where a glaring omission or patent mistake or like grave error has crept in earlier by judicial fallibility”. This has nothing to do with the basic structure violation. The Basic structure doctrine is invoked in case of an amendment to the constitution made by the parliament. This is done via Judicial Review and NOT Review petition. Hence statement 1 is not correct. As per the Civil Procedure Code and the Supreme Court Rules, any person aggrieved by a ruling can seek a review. Hence statement 2 is correct The authority of the Supreme Court to review its own judgement is provided for in Article 137 of the constitution, and not just exclusively in the Supreme court rules. Hence statement 3 is not correct. In a 2013 ruling, the Supreme Court itself laid down three grounds for seeking a review of a verdict it has delivered — the discovery of new and important matter or evidence which, after the exercise of due diligence, was not within the knowledge of the petitioner or could not be produced by him; mistake or error apparent on the face of the record; or any other sufficient reason. In subsequent rulings, the court specified that “any sufficient reason” means a reason that is analogous to the other two grounds. As per 1996 rules framed by the Supreme Court, a review petition must be filed within 30 days of the date of judgment or order. While a judgment is a final decision in a case, an order is an interim ruling that is subject to its final verdict. In certain circumstances, the court can condone a delay in filing the review petition if the petitioner can establish strong reasons that justify the delay. Review Petition is different from Judicial Review.: Judicial Review is a process under which executive or legislative actions are subject to review by the judiciary. According to Article 13(2), the Union or the states shall not make any law that takes away or abridges any of the fundamental rights, and any law made in contravention of the aforementioned mandate shall, to the extent of the contravention, be void.

Q 88.A •





Statement 1 is correct: India’s tiger population has jumped to an estimated 2,967, a rise by 33% over 2,226 reported in 2014. This is also an incredible 210% rise from 1,411 recorded in 2006, according to the all-India estimation — ‘Status of Tigers, Co-predators, Prey and their Habitat, 2018’ — released by the Prime Minister. One significant aspect of the latest estimation is the capture of 2,461 individual tigers — 83% of the total estimated 2,967 — in camera-traps. In 2014, tigers aged 1.5 years or older were counted. The current report has the cut-off age as 1 year. Statement 2 is correct: A worrying aspect of the report is the continuing loss of tiger-occupied areas. While net occupancy remains stable at 88,000-89,000 sq km, tigers relinquished over 40,000 sq km since 2014. Since they also colonised over 25,000 sq km in that period, the report computes the net loss in tiger-occupied area to be 17,881 sq km or 20% of the tiger habitat in four years. This explains the shrinking presence of tigers outside tiger reserves. Statement 3 is not correct: Against the 33% jump in the national tiger population, the report recorded potential extirpation of tigers in three reserves. No tiger was recorded in Buxa (West Bengal), Dampa (Mizoram) and Palamu (Jharkhand) tiger reserves.

Q 89.D •



29

Every two years, Forest Survey of India (FSI) undertakes assessment of country’s forest resources, the results of which are presented as the ‘India State of Forest Report (ISFR)’. Since 1987, 15 such assessments have been completed and the current assessment is the 16th in the series. The key findings of the ISFR 2019 are as follows: o The total forest cover of the country is 7,12,249 sq km which is 21.67% of the geographical area of the country. The tree cover of the country is estimated as 95,027 sq km which is 2.89% of the geographical area. www.visionias.in

Join For More Update : - https://upscpdf.com

©Vision IAS

https://t.me/UPSC_PDF

https://upscpdf.com

https://t.me/UPSC_PDF

o



The total Forest and Tree cover of the country is 8,07,276 sq km which is 24.56% of the geographical area of the country. o The current assessment shows an increase of 3,976 sq km (0.56%) of forest cover, 1,212 sq km (1.29%) of tree cover and 5,188 sq km (0.65%) of forest and tree cover put together, at the national level as compared to the previous assessment i.e. ISFR 2017. o The total forest cover in the tribal districts is 4,22,351 sq km, which is 37.54% of the geographical area of these districts. The current assessment shows a decrease of 741 sq km of forest cover within the RFA/GW in the tribal districts and an increase of 1,922 sq km outside. o The top five States in terms of increase in forest cover are Karnataka (1,025 sq km), Andhra Pradesh (990 sq km), Kerala (823 sq km), Jammu & Kashmir (371 sq km) and Himachal Pradesh (334 sq km). Hence none of the statements are correct.

Q 90.D •







Statement 2 is correct: Under the Indian Constitution, the president may also be removed before the expiry of the term through impeachment for violating the Constitution of India by the Parliament of India. Under the US Constitution, The President, Vice President and all civil Officers of the United States, shall be removed from Office on Impeachment for, and Conviction of, Treason, Bribery, or other high Crimes and Misdemeanors Statement 1 is not correct: The process of impeachment in India may start in either of the two houses of the parliament. The house initiates the process by levelling the charges against the president. The charges are contained in a notice that has to be signed by at least one-quarter of the total members of that house. The notice is sent up to the president and 14 days later, it is taken up for consideration. A resolution to impeach the president has to be passed by a two-thirds majority of the total number of members of the originating house. It is then sent to the other house. The other house investigates the charges that have been made. During this process, the president has the right to defend oneself through an authorised counsel. If the second house also approves the charges made by special majority again, the president stands impeached and is deemed to have vacated their office from the date when such a resolution stands passed. Procedure for impeaching a US President: o First, an impeachment resolution must be introduced by a member of the House of Representatives. o The speaker of the House must then direct the U.S. House Committee on the Judiciary (or a special committee) to hold a hearing on the resolution to decide whether to put the measure to a vote by the full chamber and when to hold such a vote. o A simple majority of the Judiciary Committee must approve the resolution. o If the Judiciary Committee approves the resolution, it moves to a full vote on the House floor. o If a simple majority of the those present and voting in the House approve an article of impeachment, then the president is impeached. o The procedure then moves to the Senate where a “trial” is held to determine if the president committed a crime. There is no set procedure for the trial. How it is conducted would be set by the Senate leadership. o Members of the House present evidence during the procedure. o The president would have counsel to represent him at the Senate process. o The Chief Justice of the U.S. Supreme Court presides over the trial. o Senators vote on whether the president is guilty or not guilty of the crimes he is accused of. It takes a two-thirds vote of the Senate to convict. If the president is found guilty, he is removed from office and the vice president is sworn-in as president. Statement 3 is not correct: In India, No president has faced impeachment proceedings so the above provisions have never been used. Recently, an impeachment motion has been initiated against the President of the United States (Donald Trump). So far only three presidents have been impeached in the history of the United States i.e. Bill Clinton, Andrew Johnson and Donald Trump.

Q 91.A •



30

The International Tropical Timber Organization (ITTO) is an intergovernmental organization promoting the sustainable management and conservation of tropical forests and the expansion and diversification of international trade in tropical timber from sustainably managed and legally harvested forests. ITTO: www.visionias.in

Join For More Update : - https://upscpdf.com

©Vision IAS

https://t.me/UPSC_PDF

https://upscpdf.com

https://t.me/UPSC_PDF

o



• • •

Develops internationally agreed policy guidelines and norms to encourage sustainable forest management (SFM) and sustainable tropical timber industries and trade. o Assists tropical member countries to adapt such guidelines and norms to local circumstances and to implement them in the field through projects and other activities. o Collects, analyzes and disseminates data on the production and trade of tropical timber. o Promotes sustainable tropical timber supply chains. o Helps develop capacity in tropical forestry. Forest certification is a way of verifying whether a forest area is well managed and, through labelling, of assuring consumers that in purchasing labelled products they are supporting sustainable forest management (SFM). Certification is, therefore, a tool for promoting trade in timber from sustainably managed and legally harvested forests. ITTO assists its member countries, including many small and medium-sized forest enterprises (SMEs), to pursue certification, but it remains a daunting task, especially in the tropics. Tropical forests are highly diverse, and the criteria and indicators for sustainable management (C&I) are complex; moreover, many SMEs lack the resources need to implement the exacting standards of SFM and the financial means to obtain certification for their operations. There are a total 36 Timber producing member countries and 38 timber consuming member countries. India is classified as a timber-producing member country of this organization. Hence only statements 1 and 2 are correct.

Q 92.A •



• • • • • •



Pair 1 is correctly matched: For the first time ever, the Impressed Tortoise (Manouria impressa) has been found in India. Impressed Tortoise (Manouria impressa) has been discovered in Yazali area of the Lower Subansiri district of Arunachal Pradesh. There are only two species of tortoises under the Manouria genus. India was known to be the home of only the Asian Forest Tortoise (Manouria emys) until the discovery of the Impressed Tortoise. The Asian Forest Tortoise, the largest in mainland Asia, is found only in the northeast. Its IUCN status is vulnerable. Pair 2 is not correctly matched: Golden langur – an endangered primate found in forests along the IndoBhutan border in Assam. Chakrashila is India’s first wildlife sanctuary with golden langur as the primary species. In order to ensure them the food availability, a project has been initiated under the MGNREGA to plant guava, mango, blackberry and other fruit trees in Kakoijana Reserve Forest. IUCN status is endangered. Pair 3 is correctly matched: Snow leopards live in the mountainous regions of central and southern Asia. In India, snow leopard is found along the upper reaches of the Himalayan range and, in India, it is reported to have a presence in Kashmir, Ladakh, Himachal Pradesh, Uttarakhand, Sikkim and Arunachal Pradesh. The snow leopard is listed as Vulnerable on the IUCN-World Conservation Union’s Red List of Threatened Species.

Q 93.D •







31

Statement 1 is correct: The first Global Refugee Forum (GRF), a two-day gathering of United Nations member states, was held in Geneva, Switzerland. The Forum, jointly hosted by the United Nations High Commissioner for Refugees (UNHCR), the UN Refugee Agency, and the government of Switzerland, aims to debate and discuss the response of the world’s countries to the global refugee situation. Guided by the Global Compact on Refugees, the Global Refugee Forum is an opportunity to translate the principle of international responsibility-sharing into concrete action. The Forum will showcase impactful pledges and contributions and the exchange of good practices, Statement 2 is correct: The GRF will be held every four years at the Ministerial level. It is intended to present an opportunity for UN member states and other stakeholders to announce action plans and pledges towards meeting objectives such as easing the burden on the host country, enhancing refugee selfreliance, expanding access to third-country solutions, and supporting conditions in countries of origin for return in safety and dignity. Statement 3 is not correct: India was a NOT a co-convenor of the GRF 2019 The first-ever Global Refugee Forum took place on 17 and 18 December 2019 and was a true milestone in the advancement of the international refugee regime. Co-convenor countries included Costa Rica, Ethiopia, Germany, Pakistan, Turkey and the United Nations High Commissioner for Refugees.

www.visionias.in

Join For More Update : - https://upscpdf.com

©Vision IAS

https://t.me/UPSC_PDF

https://upscpdf.com

https://t.me/UPSC_PDF

Q 94.C •



Statement 1 is correct: The generation of a database and the implementation of a web enabled Water Resources Information System popularly known as India-WRIS was initiated through a Memorandum of Understanding signed in 2008 between the Central Water Commission (CWC) and the Indian Space Research Organization (ISRO). This project was funded by the Central Water Commission. The projects has been revised and improved under the National Hydrology Project. Statement 2 is correct: It is aimed at providing upto date and reliable water data and information through web on a GIS platform in Public Domain. It is responsible for publishing the Basin reports, River Basin Atlas of India and Watershed Atlas of India.

Q 95.D •





Natural Capital includes those elements of the nature that provide valuable goods and services to humans, such as the stock of forests, food, clean air, water, land, minerals, etc. It incorporates a broad perspective on the set of services provided by ecosystems assets. Natural Capital Accounting or environmental-economic accounting is a tool that can help to gain an understanding of the interaction between the economy and the environment. It can be used to measure the state of ecosystems, flows of ecosystem services as well as changes in stocks and flows of natural resources in relation to economic changes. The natural capital accounting (NCA) method has been used to account for income and costs associated with natural resource used, based on a framework approved by the United Nations in 2012 called the System of Environmental Economic Accounts (SEEA).

Q 96.C •





Statement 1 is correct: National Mission for Clean Ganga (NMCG) was registered as a society on 12th August 2011 under the Societies Registration Act 1860. It acts as implementation arm of National Council for Rejuvenation, Protection and Management of River Ganga (referred as National Ganga Council) which was constituted under the provisions of the Environment (Protection) Act (EPA), 1986. Statement 2 is not correct: NMCG is not headed by Union Minister of Jal Shakti. It has a two tier management structure and comprises of Governing Council and Executive Committee. Both of the tiers are headed by the Director General (DG), NMCG. Statement 3 is correct: The aims and objectives of NMCG are-: o To ensure effective abatement of pollution and rejuvenation of the river Ganga by adopting a river basin approach to promote inter-sectoral co-ordination for comprehensive planning and management and o To maintain minimum ecological flows in the river Ganga with the aim of ensuring water quality and environmentally sustainable development.

Q 97.A •



• •

“Global Partnership on Wildlife Conservation and Crime Prevention for sustainable development” program also known as Global Wildlife Program (GWP) were launched in response to increasing crime against animals in natural habitat. It works towards wildlife conservation and sustainable development by fighting against illicit trafficking in wildlife through a holistic comprehensive approach. India is a partner country of the Global Wildlife Program along with other Asian and African countries. Hence statement 1 is correct. It is being implemented by the World Bank Group, United Nations Development Programme (UNDP), United Nations Environment Programme (UNEP) & Asian Development Bank (ADB). Hence statement 2 is not correct. It is funded by the Global Environment Facility (GEF). The GWP intervenes at the global, regional, and national levels. The World Bank Group’s global coordinating project establishes a learning and coordination platform to promote enhanced IWT interventions and increase technical capabilities.

Q 98.A • India has planned to install 175 GW of renewables by 2022. In this context, the Government of India has undertaken a number of policy measures for increasing share of renewable energy in India’s energy mix. These include: o Provision of Renewable Purchase Obligation (RPO) under the National Tariff Policy. It refers to the obligation imposed by law on some entities to either buy electricity generated by specified green 32

www.visionias.in

Join For More Update : - https://upscpdf.com

©Vision IAS

https://t.me/UPSC_PDF



https://upscpdf.com

https://t.me/UPSC_PDF

sources or buy, in lieu of that; renewable energy certificates from the market Hence option 1 is correct. o Permitting 100 per cent Foreign Direct Investment in the sector through automatic route. Hence option 2 is correct. o Development of Solar Parks and Ultra Mega Solar Power Projects o Development of power transmission network through Green Energy Corridor project o Making rooftop solar as a part of the housing loan provided by banks o Offshore wind energy policy for the development of offshore wind energy in the Indian Exclusive Economic Zone o Financial incentives for off-grid and decentralized renewable energy systems and devices for meeting energy needs for cooking, lighting and productive purposes The Contract for Difference (CfD) for new clean technologies o There is still no such provision in India for promoting clean/renewable sources of energy. Hence option 3 is not correct. o The contract for difference is a financial instrument offered to clean new and clean technologies to help them stay competitive in the energy markets. It provides a guaranteed price for the electricity that generators sell into the wholesale market, known as a “strike price”. When the wholesale price is below the strike price, generators are paid the difference. When it is higher, the generator pays the difference back. o It helps de-risk the technologies by offering a minimum price for energy generated and they offer this rate for an extended period. The United Kingdom provides for the Contract for Difference (CfD) to new and clean technologies.

Q 99.C •



• •

Initiated in 2014, the Integrated Tiger Habitat Conservation Programme (ITHCP) by the IUCN or ‘Tiger Programme’ is a grant-making initiative which contributes to the Global Tiger Recovery Programme (GTRP), a global effort to double tiger numbers in the wild by 2022. The programme consists of a portfolio of 12 large-scale projects in key Tiger Conservation Landscapes across Bangladesh, Bhutan, India, Indonesia, Nepal and Myanmar. The Tiger Programme is based on three pillars: o Protecting tiger species and their prey from the threat of poaching; o Preserving tiger habitats, including core habitats, buffer zones and corridors; o Supporting human populations living in tiger landscapes. The initiative is funded by The German Cooperation via KfW Development Bank, over two phases. The first phase is being implemented from 2014 to 2020 and the second one from 2019 to 2023. Hence both the statements are correct.

Q 100.C • Chabahar is the one of the ports of Iran. It consists of two separate ports namely Shahid Kalantari and Shahid Beheshti. • India and Iran agreed to develop Shahid Kalantari port in 2003. • However, it was dropped following sanctions on Iran. The project was again taken up in 2016. • Under the agreement, India will construct a 600-meter-long container handling facility at the port. India has already begun its operation at the port in 2016. • India and Iran signed the “Chabahar Project” in 2017. • It is an 85 million USD project that aims to develop Chabahar port in south-eastern Iran. The Chabahar port is an alternate route for trade between India and Afghanistan. The land route through Pakistan from India to Afghanistan is not accessible. • The project is also seen as a counter to Chinese Gwadar port in Pakistan.

Copyright © by Vision IAS All rights are reserved. No part of this document may be reproduced, stored in a retrieval system or transmitted in any form or by any means, electronic, mechanical, photocopying, recording or otherwise, without prior permission of Vision IAS. 33

www.visionias.in

Join For More Update : - https://upscpdf.com

©Vision IAS

Related Documents


More Documents from "Sanjeev kumar"